Чемпионат: Синхронный турнир "Из Минска с любовью" - 2016/17 Дата: 08-Oct-2016 - 15-Apr-2017 Тур: 1 этап Дата: 08-Oct-2016 Редактор: 1-18 - Ксения Езерская (Осиповичи - Барановичи - Минск) и Евгений Лешкович (Минск); 19-36 - Дмитрий Буценец (Пинск - Минск - Брест) Инфо: Ксения Езерская и Евгений Лешкович благодарят за тестирование вопросов и ценные замечания Ивана Топчия, Юрия Разумова, Сергея Дубелевича, Даниила Шункевича, Александра Шустера, Владислава Латынского, Дмитрия Буценца, Андрея Танану, Анну Якушевич, Ивана Зайкова, Юрия Ананича, а также команды "Ultima Ratio", "Лунь" и "На заре". Дмитрий Буценец благодарит за тестирование и помощь в подготовке пакета: Юрия Разумова, Артема Гулецкого, Марию Кленицкую, Александра Матюхина, Евгения Миротина, Ивана Зайкова, Сергея Дубелевича, Евгения Лешковича, Анастасию Балмакову, Дарью Соловей, Александру Ермалович, Михаила Карпука, а также команды "Нехорошая квартира" и "На заре" (все - Минск). Вопрос 1: Футбольный тренер Луи ван Гал уделяет большое внимание созданию комфортной атмосферы в команде. Возглавив "Манчестер Юнайтед", ван Гал первым делом распорядился поместить в клубной столовой ЕГО. Более известный ОН, по мнению историка Криса Гидлоу, располагался в 67 километрах от Манчестера. Назовите ЕГО двумя словами. Ответ: Круглый стол. Комментарий: Ван Гал распорядился поставить в столовой круглый стол, чтобы все футболисты чувствовали себя на равных. Круглый стол короля Артура, как утверждает Крис Гидлоу, располагался в городке Честер, расположенном в 66 километрах от Манчестера. Редакторы желают, чтобы комфортная атмосфера сохранялась в вашей команде на протяжении всего турнира! Источник: 1. М. Мейер. Луи ван Гал: биография. http://flibusta.is/b/454715/read 2. https://www.newsru.com/world/12jul2010/roundtable.html 3. https://www.distancecalculator.net/from-chester-to-manchester Автор: Евгений Лешкович (Минск) Вопрос 2: Министр внутренних дел Франции Мишель Альо-Мари предложила составить базу данных со всей информацией о гражданах страны: отпечатки пальцев, политические предпочтения, история семьи. За это журналисты присудили Альо-Мари неофициальную премию ЕГО. Назовите ЕГО двумя словами, начинающимися на одну и ту же букву. Ответ: Большой брат. Комментарий: Премия Большого брата ежегодно вручается лицам, "отличившимся" в наиболее грубых нарушениях неприкосновенности свободы и частной жизни. Источник: Дж. Кампфнер. Свобода на продажу. Как мы разбогатели - и лишились независимости. http://flibusta.is/b/459855/read Автор: Евгений Лешкович (Минск) Вопрос 3: [Ведущему: четко прочитать название статьи.] Статья "Швейцарский нож самочки" рассказывает, что на самом деле ОН состоит из нескольких игл, насоса и распорок. Назовите ЕГО двумя словами. Ответ: Хоботок комара. Зачет: Жало комара; комариный хоботок; комариное жало. Комментарий: Кровь пьют именно самки комаров. Хоботок на самом деле состоит из шести частей. На концах двух из них есть мелкие зубья, которыми он разрезает кожу. Через другую иглу комар вводит в кровь антикоагулянты, которые не дают крови свернуться и облегчают сосание. Еще одна игла - гибкий насос, через который происходит перекачка крови. А две другие иглы служат распорками и не дают ране закрыться. Источник: https://mi3ch.livejournal.com/3394606.html Автор: Илья Колодник (Минск) Вопрос 4: Автор статьи о Цутому Ямагути, выжившем после двух атомных бомбардировок, логично называет его "ИКСОМ". На рекламном плакате осиповичской службы такси "ИКС" можно увидеть подкову. Какое слово мы заменили ИКСОМ? Ответ: Везунчик. Комментарий: Цутому Ямагути побывал и в Хиросиме, и в Нагасаки - настоящий везунчик. Подкова - традиционный символ удачи. В названии службы такси "Везунчик" подкова заменяет букву У. Источник: 1. https://razdumyvatel.livejournal.com/12296.html 2. https://www.youtube.com/watch?v=ggwPHk3AoiA Автор: Евгений Лешкович (Минск), Ксения Езерская (Осиповичи - Барановичи - Минск) Вопрос 5: Журналист Алексей Беляков написал письмо своему другу. В нем он сообщает, что жизнь в Москве очень живая и разнообразная: для кого-то театральная, а для кого-то спортивная, для кого-то академическая, а для кого-то и пролетарская. В одном из следующих абзацев он рассказывает о девушке. Каким эпитетом он ее описывает? Ответ: Тургеневская. Зачет: Чертановская (в заметке девушка называется и так). Комментарий: Вся заметка построена на обыгрывании названий станций Московского метрополитена. "Театральная", "Спортивная", "Академическая", "Пролетарская" и "Тургеневская" - это всё станции метро. "Тургеневская барышня" - устойчивое выражение, описывающее женские манеры. Ночью девушка автора бывает "той еще Чертановской". Источник: https://www.facebook.com/permalink.php?story_fbid=1090313124358859&id=100001405462980 Автор: Павел Кебец (Барселона) Вопрос 6: Одной из достопримечательностей Яффы является плодоносящее апельсиновое дерево. По мнению ведущих передачи "Орел и решка", оно символизирует израильский народ, сумевший выжить вдали от родины. Назовите вымышленный остров, обладавший той же особенностью, что и это дерево. Ответ: Лапута. Комментарий: Это дерево парит в воздухе и не касается земли. Остров Лапута из "Путешествий Гулливера" также парил в воздухе. Летающий остров Лапута присутствует в полнометражном аниме-фильме Хаяо Миядзаки "Небесный замок Лапута". Источник: 1. "Орел и решка". 1 сезон, 7 серия. Израиль. https://www.youtube.com/watch?v=VENHncG84Eo 2. https://ru.wikipedia.org/wiki/Лапута Автор: Артур Астапенко (Минск) Вопрос 7: Дуплет. В одном из эпизодов мультсериала "Покемон" герои попадают в город, где не любят тренеров покемонов. В ресторане к ним подходит женщина и пытается выяснить их имена. Желая сохранить свою репутацию, герои поспешно придумывают себе псевдонимы. 1. Эш Кетчум представился как Том. Какую фамилию он себе придумал? 2. Тренер водных покемонов Мисти сказала, что ее зовут Анн. Какую фамилию она для себя придумала? Ответ: 1. Ато. 2. Чоус. Зачет: Всё, что напоминает правильные ответы и соответствует логике комментария/мультфильма. Комментарий: В критической ситуации ребята быстро посмотрели, что есть вокруг. А поскольку они были в ресторане, то и образовали свои псевдонимы от продуктов питания. Источник: Мультсериал "Покемон: Лига Индиго", серия "Тайна Мрачного города". Автор: Павел Кебец (Барселона) Вопрос 8: В одном из эпизодов мультсериала "Человек-паук" главный герой ведет в Москве суровый бой с шестью суперзлодеями. Один из них предлагает сыграть в ТАКОЙ ИКС, отводя Человеку-пауку роль АЛЬФЫ. Примечательно, что обычно в ТАКОМ ИКСЕ используют не АЛЬФУ, да и для комплекта необходимо почти вдвое больше суперзлодеев. Что мы заменили словами "ТАКОЙ ИКС"? Ответ: Русский хоккей. Зачет: Бенди; хоккей с мячом. Комментарий: ТАКОЙ ИКС - русский хоккей, АЛЬФА - шайба. В русский хоккей, он же бенди, он же хоккей с мячом, играют мячом. В команде по хоккею с шайбой играет шесть человек, в русском хоккее - одиннадцать. Слова "ведет суровый бой" являются намеком на песню о том, как "суровый бой ведет ледовая дружина". Источник: 1. Мультсериал "Человек-Паук" (1994-1997), 55-я серия "Шесть забытых воинов 2: Невостребованное наследие", примерно 9-я - 10-я минуты. 2. https://ru.wikipedia.org/wiki/Хоккей_с_мячом Автор: Павел Кебец (Барселона) Вопрос 9: Программы распознавания изображений увидели в ИКСАХ "чудовище", "птицу, летящую над часами", "ракушку" и даже "орхидею". ИКСЫ героя экранизированного комикса появлялись из двух жидкостей, помещенных между слоями латекса. Какие два слова, начинающиеся на соседние буквы, мы заменили ИКСАМИ? Ответ: Пятна Роршаха. Источник: 1. https://geektimes.ru/post/276980/ 2. https://ru.wikipedia.org/wiki/Роршах_(Хранители) Автор: Илья Колодник (Минск) Вопрос 10: В 1980-е годы финский футбольный клуб "Куусюси" славился надежной игрой в защите. Какая фамилия фигурировала в прозвище, которое дали этому клубу журналисты? Ответ: Маннергейм. Комментарий: Журналисты сравнивали непроходимую оборону этой команды со знаменитой линией Маннергейма. Источник: https://www.sports.ru/tribuna/blogs/footballeshkovich/411593.html Автор: Евгений Лешкович (Минск) Вопрос 11: Рассказывая о сцене рисования, Ольга Суркова упоминает обгоревшую головку космического корабля и алюминиевый пол. Первое рисование совершила экспедиция Шеннона. Какие семь букв мы дважды пропустили в этом вопросе? Ответ: присоля. Комментарий: Так по аналогии с приземлением назвали сцену посадки на Солярис члены съемочной группы фильма Андрея Тарковского. Источник: 1. http://kinoart.ru/archive/2002/04/n4-article16 2. С. Лем. Солярис. http://flibusta.is/b/232728/read Автор: Александр Шустер (Минск) Вопрос 12: В 1715 году Петр I издал указ, согласно которому любой крепостной, совершивший донос на своего барина, получал вольную. По оригинальной версии журналистки Лилии Гущиной, именно тогда русские дворяне начали ДЕЛАТЬ ЭТО. Мать Наполеона Бонапарта ДЕЛАТЬ ЭТО так и не научилась. Что мы заменили словами "ДЕЛАТЬ ЭТО"? Ответ: Говорить по-французски. Зачет: По словам "говорить" и "французский". Комментарий: Как нетрудно догадаться, после петровского указа по России прокатилась волна доносов. Чтобы успешнее "шифроваться" от слуг, русские дворяне стали говорить по-французски. Мать Наполеона Бонапарта была корсиканской националисткой и до конца жизни так и не выучилась говорить по-французски. Источник: 1. Журнал "Story", 1994, N 8. - С. 8. 2. https://ru.wikipedia.org/wiki/Рамолино,_Мария_Летиция Автор: Евгений Лешкович (Минск) Вопрос 13: Поэт и живописец XVI века Карел ван Мандер написал биографии многих современников: Рогира ван дер Вейдена, Иеронима Босха, Питера Брейгеля-старшего. Искусствовед Паола Волкова называет ван Мандера ТАКИМ ИМ. Какие два слова, которые начинаются на соседние буквы алфавита, мы заменили словами "ТАКОЙ ОН"? Ответ: Голландский Вазари. Комментарий: Карел ван Мандер сделал то же, что и его куда более известный коллега Джорджо Вазари - автор "Жизнеописаний прославленных живописцев, скульпторов и архитекторов". Источник: 1. https://ru.wikipedia.org/wiki/Мандер,_Карел_ван 2. П.Д. Волкова. Мост через бездну. Мистики и гуманисты. http://flibusta.is/b/454040/read Автор: Евгений Лешкович (Минск) Вопрос 14: <раздатка> Deep View Из-за сложного рельефа Фарерские острова очень долго не были представлены в проекте Google Street View. Как правило, панорамная съемка в рамках этого проекта проводится с помощью автомобилей, но на Фарерах это технически невозможно. Решение нашли местные жители, которые организовали кампанию "Deep View". Какие две буквы в этом вопросе мы заменили одной? Ответ: Sh. Комментарий: Фарерцы догадались приделывать камеры к овцам, которых на островах в два раза больше, чем людей. Собственно, само название "Фареры" означает "Овечьи острова". Источник: Журнал "GEO", 2016, N 9. - С. 26. Автор: Евгений Лешкович (Минск) Вопрос 15: Джон Кампфнер пишет, что на время Олимпиады-2008 в пекинских киосках прекратилась продажа пива, вина и джина. В объяснении причин такого запрета упоминается фамилия уроженца Вятской губернии. Назовите эту фамилию. Ответ: Молотов. Комментарий: Официальный Пекин боялся, как бы во время Игр не случилось демонстраций и уличных столкновений, поэтому отказался от продажи напитков в стеклянной таре. Это позволило не бояться так называемых "коктейлей Молотова". Вячеслав Молотов родился в слободе Кукарка Вятской губернии. Источник: 1. Дж. Кампфнер. Свобода на продажу. Как мы разбогатели - и лишились независимости. http://flibusta.is/b/459855/read 2. https://ru.wikipedia.org/wiki/Молотов,_Вячеслав_Михайлович Автор: Евгений Лешкович (Минск) Вопрос 16: Герой Тома Хэнкса неудачно провел презентацию в Саудовской Аравии. Его дочь шутит, что за это его ОТЧИТАЛИ. Какое слово мы заменили словом "ОТЧИТАЛИ"? Ответ: Пропесочили. Комментарий: Королевство Саудовская Аравия, как известно, находится в пустыне. Источник: Х/ф "Голограмма для короля" (2016), реж. Том Тыквер. Автор: Александр Шустер (Минск) Вопрос 17: После успешного клонирования овечки Долли генетик Артур Каплан получил несколько одинаковых предложений, на которые ответил: "Вы хотите помочь единственному, кто может отлично справиться без нас!". Какой город упоминался во всех этих предложениях? Ответ: Турин. Комментарий: Религиозные фанатики зажглись идеей клонировать Христа. По мнению Каплана, это глупо, потому что Иисус, если верить Новому завету, без проблем может воскреснуть и без помощи генетиков. По канонической версии, ДНК Христа содержится на туринской плащанице. Источник: С. Кин. Синдром Паганини и другие правдивые истории о гениальности, записанные в нашем генетическом коде. http://flibusta.is/b/426306/read Автор: Евгений Лешкович (Минск) Вопрос 18: В 2014 году белорусский программист Геннадий Короткевич победил в нескольких престижных турнирах по спортивному программированию: Google Code Jam, Facebook Hacker Cup, TopCoder Open и Яндекс: Алгоритм. Согласно шутке одного журналиста, Геннадий выиграл программистский ИКС. Какие два слова мы заменили ИКСОМ? Ответ: Большой шлем. Комментарий: Прямая аналогия с теннисом, где Большой шлем можно получить, выиграв четыре крупнейших турнира. Источник: 1. http://www.russoft.ru/tops/2157 2. https://ru.wikipedia.org/wiki/Турниры_Большого_шлема_(теннис) Автор: Павел Кебец (Барселона) Вопрос 19: Романо Муссолини вспоминал, что его отец рассчитывал с небольшим количеством людей добраться до Вальтеллины в Альпах и сопротивляться там до последнего. Бенито Муссолини сравнивал Вальтеллину с НИМИ. Назовите ИХ. Ответ: Фермопилы. Комментарий: История напоминает ситуацию во время сражения в Фермопилах и историю о трехстах спартанцах. Источник: Р. Муссолини. Дуче, мой отец. Автор: Никита Шевела (Минск) Вопрос 20: Персонажи Самюэля Беккета замечают, что ОНА не плачет. После этого один из них предполагает, что ОНА умерла. Назовите ЕЕ коротким словом. Ответ: Ива. Комментарий: Плакучая. Источник: С. Беккет. В ожидании Годо. https://books.google.ru/books?id=B91TKeLQ54EC&pg=PT30#v=onepage&q&f=false Автор: Дмитрий Буценец (Пинск - Минск - Брест) Вопрос 21: Джордж Байрон, описывая сражение, пишет, что на месте погибших солдат тут же появлялись другие. Ответьте одним словом: с кем поэт сравнил двигающиеся по берегу полкИ? Ответ: Гидра. Комментарий: Гидра - змееподобное чудовище из греческой мифологии, чье имя можно перевести как "водяная змея". Байрон по этому поводу писал: "Пускай героев головы слетают - // Немедленно другие вырастают". Источник: Дж. Байрон. Дон Жуан. http://flibusta.is/b/424289/read Автор: Дмитрий Буценец (Пинск - Минск - Брест) Вопрос 22: Тэнгу - существо из японских поверий - летает при помощи НЕГО. Персонажи одного романа использовали ЕГО, чтобы переместить обездвиженного призрака. Назовите ЕГО одним словом. Ответ: Веер. Комментарий: Несмотря на то что у тэнгу есть крылья, летает он именно с помощью веера. Веера - неотъемлемая часть японской культуры. Для перемещения бесплотного призрака герои Роулинг использовали ветер от огромного веера. Источник: 1. https://ru.wikipedia.org/wiki/Тэнгу 2. Дж.К. Роулинг. Гарри Поттер и Тайная комната (перевод М.Д. Литвиновой). http://flibusta.is/b/265596/read Автор: Дмитрий Буценец (Пинск - Минск - Брест) Вопрос 23: Артем Заяц рассказывает, как при показе "Прибытия поезда" люди в ужасе выбегали из зала, и отмечает, что этот поезд СДЕЛАЛ ЭТО. Ответьте тремя словами: что сделал? Ответ: Сломал четвертую стену. Зачет: Разрушил четвертую стену и т.п. Комментарий: Разрушение четвертой стены - это прямое обращение героя художественного произведения к автору или аудитории. Фильм "Прибытие поезда" заставил многих зрителей в ужасе выбежать из зала. Источник: https://www.film.ru/articles/kinoslovar-chetvertaya-stena Автор: Дмитрий Буценец (Пинск - Минск - Брест) Вопрос 24: В цикле "Темная башня" Стивен Кинг неоднократно ломает четвертую стену. Персонажи цикла замечают, что порой спасались в последний момент, и сравнивают писателя с НИМ. Назовите ЕГО тремя словами. Ответ: Бог из машины. Зачет: Deus ex machina. Комментарий: "Бог из машины" - выражение, означающее неожиданную, нарочитую развязку той или иной ситуации, с привлечением внешнего, ранее не действовавшего в ней фактора. Источник: С. Кинг. Темная башня. https://books.google.ru/books?id=dcy5AAAAQBAJ&pg=PT167#v=onepage&q&f=false Автор: Дмитрий Буценец (Пинск - Минск - Брест) Вопрос 25: Персидская легенда называет деревню около ЭТОГО ОБЪЕКТА колыбелью человечества. В курдском названии ЭТОГО ОБЪЕКТА есть слово "огненная". Под каким названием ЭТОТ ОБЪЕКТ известен нам? Ответ: Арарат. Комментарий: Курдское название горы свидетельствует о ее вулканическом происхождении. Согласно персидской легенде, считалось, что деревня Аргури на склоне Арарата находится на месте, где Ной построил алтарь и посадил первый виноградник. Источник: 1. https://www.britannica.com/place/Mount-Ararat 2. https://ru.wikipedia.org/wiki/Арарат Автор: Дарья Данилевич (Минск), Дмитрий Буценец (Пинск - Минск - Брест) Вопрос 26: Персонаж фильма "Приключение" говорит, что людьми, которые ежегодно пропадают в Италии, можно заполнить ЕГО. В оформлении музея при НЕМ преобладают красный, черный и синий цвета. Назовите ЕГО. Ответ: [Стадион] "Сан-Сиро". Зачет: [Стадион] "Джузеппе Меацца". Комментарий: Персонаж фильма говорит, что людьми можно заполнить стадион Сан-Сиро. Стадион является домашним для двух клубов - "Милана" и "Интера". Их цвета - красно-черные и сине-черные соответственно. Источник: 1. Х/ф "Приключение" (1960), реж. Микеланджело Антониони. 2. https://ru-travel.livejournal.com/13418232.html Автор: Дмитрий Буценец (Пинск - Минск - Брест) Вопрос 27: Описывая ЭТОТ ПРОЦЕСС, высадившийся на берег персонаж Милорада Павича образно говорит, что рыбьи кости постепенно обрастали мясом. Назовите ЭТОТ ПРОЦЕСС. Ответ: Постройка корабля. Зачет: Синонимичные ответы. Источник: М. Павич. Пейзаж, нарисованный чаем. http://flibusta.is/b/41905/read Автор: Дмитрий Буценец (Пинск - Минск - Брест) Вопрос 28: Название древнего города Нан-МадОл, расположенного в Микронезии, в переводе означает "промежутки". Какой европейский топоним упоминается в его прозвище? Ответ: Венеция. Комментарий: Город расположен на искусственном архипелаге из 92 островов, связанных системой "промежутков" - искусственных каналов. Источник: https://ru.wikipedia.org/wiki/Нан-Мадол Автор: Никита Шевела (Минск), Дмитрий Буценец (Пинск - Минск - Брест) Вопрос 29: В Великобритании даже отдельные сцены впервые были показаны только спустя двадцать лет в документальном фильме "Запретный плод". Какому фильму был посвящен "Запретный плод"? Ответ: "Заводной апельсин". Комментарий: Несмотря на протесты режиссера, кадры из фильма были использованы. Запретный плод "апельсина" оказался сладок. Источник: https://en.wikipedia.org/wiki/A_Clockwork_Orange_(film) Автор: Дмитрий Буценец (Пинск - Минск - Брест) Вопрос 30: <раздатка> | Год Живые языки | 2015 ~46 млн. ~6800 | 2050 ~131 млн. ~3400 Во втором столбце можно увидеть данные, которые приводят нейробиологи Мартин Принц и Андерс ВИмо. Ответьте двумя словами, начинающимися на соседние буквы алфавита: распространение чего он показывает? Ответ: Болезнь Альцгеймера. Комментарий: Существуют серьезные статистические научные исследования на тему того, что владение несколькими языками препятствует или как минимум затормаживает развитие болезни Альцгеймера. Сокращение живых языков, скорее всего, будет способствовать тому, что люди реже будут знать несколько языков одновременно. Источник: 1. http://miresperanto.com/aliaj_lingvoj/najdjot.htm 2. http://www.vesti.ru/doc.html?id=1152674 3. http://medportal.ru/mednovosti/news/2015/08/26/745dementia/ Автор: Никита Шевела (Минск) Вопрос 31: Словами "ИКС" и "ИГРЕК" заменены другие слова. Среди персонажей одной манги есть русский боец. Одна из его атак - комбинация ударов ладонью и кулаком носит название "ИКС и ИГРЕК". Что сделал персонаж другого произведения после отправления из "ИКСА и ИГРЕКА"? Ответ: И немедленно выпил. Зачет: Немедленно выпил; выпил. Комментарий: ИКС и ИГРЕК - это серп и молот. Произведение - "Москва - Петушки". Речь идет о главе "Серп и молот - Карачарово", которая состоит из одного предложения: "И немедленно выпил". Источник: 1. http://mangachan.me/online/105658-strongest-disciple-kenichi_v32_ch297.html#page=13 2. В.В. Ерофеев. Москва - Петушки. http://flibusta.is/b/282345/read Автор: Никита Шевела (Минск) Вопрос 32: Внимание, в вопросе словом "ПРОПУСК" мы заменили два слова. В своем труде Фрейд рассуждал о кажущейся пассивности в создании сновидений. В качестве примера он приводил часто произносимые нами слова "ПРОПУСК". В русскоязычной песне после слов "ПРОПУСК" упоминается то, что можно увидеть в серии работ известного француза. Что же именно? Ответ: Небо Лондона. Комментарий: Фрейд отмечает в словах "мне приснилось" пассивный контекст. Известный француз - Клод Моне, а серия - "Здание парламента в Лондоне". Источник: 1. З. Фрейд. Толкование сновидений. https://archive.org/stream/Freud_1900_Die_Traumdeutung_k#page/32/mode/2up 2. http://www.karaoke.ru/artists/zemfira/text/london/ 3. https://en.wikipedia.org/wiki/Houses_of_Parliament_(Monet_series) Автор: Ренат Рустамов (Минск) Вопрос 33: 10 января 1912 года, когда до цели оставалось около 75 миль, был заложен склад, который получил название "Полтора градуса". Назовите эту цель. Ответ: Южный полюс. Комментарий: В подобном духе называла свои склады группа Роберта Скотта. Координаты южного полюса - 90°00'00" южной широты. Долготы, как известно, полюс не имеет, чем и обусловлена идея названия складов. Источник: https://ru.wikipedia.org/wiki/Скотт,_Роберт Автор: Дмитрий Буценец (Пинск - Минск - Брест) Вопрос 34: Ребенок из книги Эммы Донохью всю жизнь прожил взаперти вдвоем с мамой. Когда мальчик впервые говорит с другими людьми о матери, ему не сразу удается использовать ЭТО. На известном объекте ИМ можно назвать Теодора Рузвельта. Назовите ЭТО двумя словами. Ответ: Третье лицо. Комментарий: Мальчику не сразу удается сказать о матери "она", вместо этого он дважды произносит "ты". Гора Рашмор известна своим скульптурным барельефом четырех президентов США. Если смотреть слева направо, "третьим лицом" будет Теодор Рузвельт. Источник: 1. Э. Донохью. Комната. https://books.google.ru/books?id=Kw9aDwAAQBAJ&pg=PT196#v=onepage&q&f=false 2. https://ru.wikipedia.org/wiki/Рашмор Автор: Оксана Бандура (Минск) Вопрос 35: На картине конца восемнадцатого века мужчина играет в шахматы. Описывая прическу мужчины - по бокам волосы длинные, а сзади подстрижены коротко, - Википедия отмечает, что такая прическа могла облегчить ЕГО работу. Назовите ЕГО. Ответ: Палач. Комментарий: Такая мода появилась во времена французской революции. Модники демонстративно показывали свое неприятие революции. Перед гильотинированием палач сзади состригал волосы. На картине используется известный сюжет игры в шахматы со смертью. Источник: https://ru.wikipedia.org/wiki/Портрет_доктора_де_С.,_играющего_в_шахматы_со_Смертью Автор: Дмитрий Буценец (Пинск - Минск - Брест) Вопрос 36: В противостоянии ИКСА остальным Поль ВалерИ видел символ разумного сознания. Малайцы называют ИКС старшим братом. Какие два слова мы заменили ИКСОМ? Ответ: Большой палец. Комментарий: В биологии термин звучит как "противопоставленный большой палец". В результате эволюции это стало характерно только для человека и некоторых других приматов. Источник: 1. М. Турнье. Зеркало идей. http://flibusta.is/b/313487/read 2. http://russian7.ru/post/o-chem-mozhet-rasskazat-bolshoj-palec/ Автор: Ренат Рустамов (Минск), Дмитрий Буценец (Пинск - Минск - Брест) Тур: 2 этап Дата: 12-Nov-2016 Редактор: 1-18 - Валерий Семёнов (Минск - Могилев); 19-36 - Вадим Кузмич (Логойск) Инфо: Валерий Семёнов благодарит команду "Дятлы" за присланные вопросы, а команду "Бренно лаская" за тестирование пакета. Вадим Кузмич благодарит Даниила Шункевича, Андрея Танану, Аню Якушевич, Виталия Калачёва, Евгения Лешковича за помощь в подготовке пакета и ценные замечания, команды "New-реанимация" и "Aspers" за присланные вопросы, команду "Бренно лаская" за тестирование пакета. Вопрос 1: На карикатуре Жана Эффеля ЕЕ поздравляют с удачным выбором, ведь лучшего человека на свете не найти. Назовите ЕЕ. Ответ: Ева. Комментарий: Лучшим мужем на свете был Адам. Жан Эффель известен своей серией рисунков "Сотворение мира". Источник: Ж. Эффель. Сотворение мира. - М.: Изобразительное искусство, 1989. - С. 554. Автор: Сергей Кушмар (Минск) Вопрос 2: Этнолог Джаред Даймонд пишет, что для высокогорных племен Новой Гвинеи в силу отсутствия у них домашних животных был характерен низкобелковый рацион. Так ученый объясняет широкую распространенность ЭТОГО среди новогвинейских племен. Назовите ЭТО одним словом. Ответ: Людоедство. Зачет: Каннибализм. Комментарий: Больше белок взять негде. Источник: Дж. Даймонд. Ружья, микробы и сталь. Судьбы человеческих обществ. http://flibusta.is/b/237834/read Автор: Валерий Семёнов (Минск - Могилев) Вопрос 3: Саудовский экономист Абдалла Муса бен Тайр рассказывает, что обострение политического конфликта в 2014 году для его страны не было критичным, так как в то время ОН состоял преимущественно из фисташек, шафрана и ковров. Назовите ЕГО тремя словами, начинающимися на одну и ту же букву. Ответ: Импорт из Ирана. Комментарий: Разрыв отношений с Ираном экономика Саудовской Аравии не почувствовала, чего нельзя сказать об иранской экономике. Источник: https://meduza.io/feature/2016/01/11/eto-voyna-kak-vnutri-strany-tak-i-za-ee-predelami Автор: Валерий Семёнов (Минск - Могилев) Вопрос 4: Игорь Малевич был одним из первых советских ученых, командированным в НАСА. По его словам, когда он впервые появился в НАСА, у него осторожно спросили, не умер ли у него кто-нибудь. Какие слова мы пропустили в вопросе? Ответ: В костюме. Зачет: В черном костюме; в деловом костюме; в галстуке. Комментарий: В НАСА все ходили в свободной одежде, обычно в джинсах и футболке. Источник: https://www.kp.by/daily/23873.3/144169/ Автор: Валерий Семёнов (Минск - Могилев) Вопрос 5: После Всемирной Парижской выставки 1900 года пресса назвала ЕЕ "позвоночным хребтом русского великана". Назовите ЕЕ максимально точно. Ответ: Транссибирская магистраль. Зачет: Транссиб; Сибирская железная дорога. Комментарий: Считалось, что Транссиб обеспечит России приоритет во всём Азиатско-Тихоокеанском регионе, а Маньчжурию превратит в русскую провинцию. На выставке 1900 года Транссибирская магистраль была представлена общественности. Источник: Изобретение века. Проблемы и модели времени в России и Европе XIX столетия. - М.: Новое литературное обозрение, 2013. - С. 248. Автор: Элеонора Кутас, Сергей Кушмар (Минск) Вопрос 6: Кирилл Бабаев пишет, что от инфекций, вызванных ржавыми ритуальными ножами и грязной "священной" водой, в Африке продолжают умирать тысячи инфицированных детей. В предыдущем предложении мы немного изменили одно слово. Восстановите это слово. Ответ: Инициированных. Комментарий: Обряд инициации у многих народов связан с обрезанием или ритуальными надрезами, в результате которых можно получить заражение крови. Источник: http://prochtenie.ru/passage/28336 Автор: Валерий Семёнов (Минск - Могилев) Вопрос 7: Внимание, в вопросе есть замена. Статья в журнале "Вокруг света" об ЭТОМ ГОРОДЕ называлась "Флоридские бордюры". Назовите ЭТОТ ГОРОД. Ответ: Сент-Питерсберг. Зачет: Санкт-Петербург. Комментарий: Статья называлась "Флоридские поребрики". Источник: http://www.vokrugsveta.ru/article/242489/ Автор: Сергей Кушмар (Минск) Вопрос 8: Игорь Можейко пишет, что правивший во второй половине XII века император Ши-Цзун восстановил Великий шелковый путь, и в китайских торговых городах стала слышна монгольская, арабская, персидская и даже ТАКАЯ речь. Какая - ТАКАЯ? Ответ: Итальянская. Комментарий: Чуть позже туда уже по проторенной другими купцами дорожке вроде как доехал Марко Поло. Источник: И.В. Можейко. 1185 год. Восток - Запад. Истоки. Мир ислама. Между двух миров. http://flibusta.is/b/418253/read Автор: Валерий Семёнов (Минск - Могилев) Вопрос 9: В покере комбинацию из девятки и короля иногда называют двумя словами, начинающимися на соседние буквы алфавита. Какими? Ответ: Собачья работа. Комментарий: Название комбинации намекает на название фильма "К-9: Собачья работа". Источник: https://ru.wikipedia.org/wiki/К-9_(фильм) Автор: Анна Ермачёнок (Минск) Вопрос 10: Распутная римлянка из романа Генриха Сенкевича рассказала о сне, в котором видела себя ЕЮ. Это вызвало всеобщий смех. Назовите ЕЕ одним словом максимально точно. Ответ: Весталка. Комментарий: В романе описывается Рим первого века нашей эры. Весталки должны были вести целомудренный образ жизни. Источник: 1. Г. Сенкевич. Камо грядеши. http://flibusta.is/b/279412/read 2. http://ru.wikipedia.org/wiki/Весталки Автор: Дмитрий Вайтович (Минск) Вопрос 11: В 1973 году житель Кингстона Роберт Марлин заявил, что в целях самозащиты застрелил некоего Джона Брауна. Назовите профессию Джона Брауна. Ответ: Полицейский. Зачет: Шериф. Комментарий: Этот случай послужил вдохновением Бобу Марли для написания песни "I Shot The Sheriff". Источник: https://song-story.ru/i-shot-sheriff-bob-marley/ Автор: Михаил Басс (Ярцево) Вопрос 12: Работая в Корнелльском университете, Ричард Фейнман получил предложение из Калифорнийского Технологического института. Корнелл не хотел терять Фейнмана и поэтому сделал ему встречное предложение. Но стоило Фейнману надумать остаться в Корнелле, как поступало новое предложение от Калтеха, и так далее. Описывая эти события, Фейнман упоминает ЕГО, столкнувшегося с дополнительными трудностями. Назовите ЕГО. Ответ: [Буриданов] осел. Комментарий: Эту ситуацию Фейнман уподобил истории с ослом, стоящим между двумя охапками сена, но только столкнувшимся с дополнительным затруднением: стоит ему шагнуть к одной, как другая вырастает в размерах. Источник: Р. Фейнман. "Вы, разумеется, шутите, мистер Фейнман!". http://flibusta.is/b/497060/read Автор: Дмитрий Вайтович (Минск) Вопрос 13: Обычный американский астроном может поработать с телескопом Хейла всего несколько смен в году. Поэтому во время своей смены астрономы надеются, что будет ОНА. "ОНА" - название произведения 1889 года. Назовите ЕЕ. Ответ: Звездная ночь. Комментарий: Ты около года ждешь своей очереди, поэтому очень обидно, если небо в твою смену закрыто облаками. Источник: 1. М. Браун. Как я убил Плутон и почему это было неизбежно. http://flibusta.is/b/297958/read 2. https://ru.wikipedia.org/wiki/Звёздная_ночь Автор: Валерий Семёнов (Минск - Могилев) Вопрос 14: Однажды в разговоре с молодым марксистом Энгельс заявил: "Надо, чтобы русские поменьше ПРОПУСК и Маркса, а мыслили так, как это делал бы Маркс на их месте". В этом вопросе мы ПРОПУСК. Какие два слова мы пропустили? Ответ: Цитировали Энгельса. Комментарий: Приведенная нами цитата из Энгельса звучит так: "Надо, чтобы русские поменьше цитировали Энгельса и Маркса, а мыслили так, как это делал бы Маркс на их месте". Источник: http://propaganda-journal.net/834.html Автор: Валерий Семёнов (Минск - Могилев) Вопрос 15: Добраться в Норильск можно либо воздушным транспортом, либо водным, поэтому в Норильске остальную часть России называют "ИКСОМ". Назовите тот ИКС, который в 2015-2016 годах посетило около 40 тысяч туристов. Ответ: Антарктида. Комментарий: ИКС - это материк. Источник: 1. https://ru.wikipedia.org/wiki/Норильск 2. http://2.russia.tv/article/show/article_id/208/ 3. http://www.iaato.org/tourism-statistics Автор: Сергей Кушмар (Минск) Вопрос 16: Дуплет. 1. Председатель Китайской Народной Республики Си Цзиньпин - страстный поклонник футбола. После переговоров в Германии по вопросам фармакологии ему подарили ЕЕ. Назовите ЕЕ двумя словами. 2. Председатель Китайской Народной Республики Си Цзиньпин - страстный поклонник футбола. После переговоров в Германии по вопросам автомобилестроения ему подарили ЕЕ. Назовите ЕЕ двумя словами, начинающимися на парные согласные. Ответ: 1. Футболка "Байера". 2. Футболка "Вольфсбурга". Комментарий: (pic: 20161019.jpg) Основные спонсоры этих команд - крупные фармакологическая и автомобилестроительная компании соответственно. К слову, спонсор "Вердера" - крупная аграрная компания, не имеющая отношения к автомобилестроению. Источник: https://www.sports.ru/tribuna/blogs/sportbizinfo/850776.html Автор: Валерий Семёнов (Минск - Могилев) Вопрос 17: Персонаж Леонида Андреева узнаёт, что его спутница, внешне безгрешная, является распутной женщиной. Каждый вечер он бил ее, однако каждое утро, забыв вчерашнее, он начинал верить ей снова. Назовите двумя словами, начинающимися с одной буквы, то, с чем персонаж сравнивал свою веру. Ответ: Печень Прометея. Комментарий: "Как печень у Прометея, за ночь вырастала моя вера, и как коршун, целый день она терзала ее". Источник: Л.Н. Андреев. Дневник Сатаны. http://az.lib.ru/a/andreew_l_n/text_0850.shtml Автор: Дмитрий Вайтович (Минск) Вопрос 18: В одном российском романе утверждается, что для надежности ЕГО следует запить канистрой самогона. Назовите ЕГО двумя короткими словами. Ответ: Пуд соли. Комментарий: Цитата из романа: "Пословица про пуд соли права, но, на мой взгляд, эти шестнадцать кило надо еще запить канистрой самогонки - вот тогда точно поймешь, с кем имеешь дело". Источник: Д. Донцова. Тормоза для блудного мужа. https://books.google.ru/books?id=TLCvIma27FMC&pg=PT41#v=onepage&q&f=false Автор: Ольга Ярошенко (Могилев) Вопрос 19: В 2013 году организаторы юбилейной велогонки "Тур де Франс" имели грандиозные планы и решили провести начало гонки в новом для велогонки регионе. Назовите этот регион. Ответ: Корсика. Комментарий: Планы ну просто наполеоновские. Наполеон, как известно, родился на Корсике. Источник: 1. https://fr.wikipedia.org/wiki/Tour_de_France_2013 2. Журнал "Discovery", 2015, N 8 (78). - С. 40. Автор: Владимир Чернушкин (Минск) Вопрос 20: Скульптор, персонаж сказки Петра Бормора, использует ЕЕ для создания максимально реалистичных скульптур. Назовите ЕЕ двумя словами, начинающимися на одну и ту же букву. Ответ: Голова Горгоны. Комментарий: Скульптор усаживал натурщицу в нужную ему позу, после чего показывал ей голову Медузы Горгоны, и натурщица обращалась в каменную скульптуру. Источник: П. Бормор. Книга на третье. http://flibusta.is/b/191906/read Автор: Павел Новиков (Витебск - Минск) Вопрос 21: Иоханнес Шефферус предполагал, что скандинавские языческие боги Тор, Один и Фрейя символизируют ЭТО. Назовите ЭТО двумя словами. Ответ: Тре Крунур. Зачет: Три короны. Комментарий: Тре Крунур (Три короны) - национальный геральдический символ Швеции. Источник: https://ru.wikipedia.org/wiki/Три_короны Автор: Андрей Танана (Минск) Вопрос 22: Внимание, в вопросе есть замены. Эрик Вейнер рассказывает, что видел в городе Ханчжоу автобус с номером KI55 [кей-ай пятьдесят пять]. Местные жители называют этот автобус ТАКИМ. В названии какой российской достопримечательности есть слово "ТАКОЙ"? Ответ: Поцелуев мост. Комментарий: KI55 очень похоже на английское слово "kiss" [кисс], поэтому жители Ханчжоу называют свой автобус "поцелуйным". Поцелуев мост находится в Санкт-Петербурге. Источник: 1. Э. Вейнер. География гениальности. Где и почему рождаются великие идеи. http://flibusta.is/b/465171/read 2. https://ru.wikipedia.org/wiki/Поцелуев_мост_(Санкт-Петербург) Автор: Евгений Лешкович (Минск) Вопрос 23: В конце 2000 года на создание самого большого ЕГО из мороженого пошло около 300 килограммов продукта: сливочного, черничного и клубничного. В одном из слов этого вопроса мы пропустили несколько букв. Напишите это слово в первоначальном виде. Ответ: Снеговика. Комментарий: Канун Нового года как-никак. Классический снеговик состоит из трех шаров, для каждого из которых использовали свой тип мороженого. Источник: https://www.newsru.com/russia/31dec2000/icecream.html Автор: Вадим Кузмич (Логойск) Вопрос 24: Дуплет. 1. Когда илионцы вышли на берег, то увидели, что дар Богам, оставленный там, никуда не годится. Какие шесть слов мы заменили в этом вопросе? 2. За последние две тысячи лет ЭТО уменьшилось в размере в несколько раз. В статье Википедии об ЭТОМ упоминаются, в частности, канаты, сплетенные из верблюжьего волоса. Назовите ЭТО двумя словами. Ответ: 1. Ни в какие ворота не лезет. 2. Игольное ушко. Комментарий: 2. По одной из версий, самые маленькие городские ворота Иерусалима носили название "Игольное ушко". Верблюд мог пройти в них только без поклажи и наездника. Отсюда и происхождение известной фразы. Источник: 1. http://история-вещей.рф/instrumentyi/shveynyie-iglyi.html 2. https://ru.wikipedia.org/wiki/Игольное_ушко Автор: Александр Санин (Минск) Вопрос 25: В 1979 году погиб Роберт Уильямс, рабочий завода Ford Motor Co. [форд мОтор кАмпани]. Это был первый официально зарегистрированный случай нарушения закона. Наказания, впрочем, не последовало, так как самим Законом оно не предусмотрено. Назовите этот закон абсолютно точно. Ответ: Первый закон робототехники. Комментарий: Персонаж книги "Я робот" погибает от рук робота, тем самым нарушившего первый закон робототехники, запрещающий причинять вред человеку. Источник: 1. https://en.wikipedia.org/wiki/Robert_Williams_(robot_fatality) 2. https://en.wikipedia.org/wiki/Three_Laws_of_Robotics Автор: Александр Санин (Минск) Вопрос 26: Действие компьютерной игры "Skyrim" [скАйрим] происходит в вымышленной северной провинции. По сюжету одного из заданий герою поручено выкрасть родословную знаменитого скакуна. Кто значится в родословной самым ранним предком этого скакуна? Ответ: Слейпнир. Комментарий: Игра имеет множество аллюзий на историю и мифологию Скандинавии. В частности, знаменитый скакун ведет свою родословную от самого коня Одина. Источник: http://ru.elderscrolls.wikia.com/wiki/Мороз Автор: Даниил Шункевич (Минск) Вопрос 27: По мнению героя одной книги, неприятный запах в автомобиле не смог бы замаскировать целый ИКС освежителей воздуха. Назовите ИКС коротким словом. Ответ: Лес. Комментарий: Вонь стояла такая, что даже лес освежителей воздуха не смог бы ее замаскировать. В автомобилях широко распространены освежители воздуха в виде елочек. Источник: Р. Риггз. Дом странных детей. http://flibusta.is/b/299551/read Автор: Вадим Кузмич (Логойск) Вопрос 28: (pic: 20161020.jpg) На розданном шуточном изображении мы скрыли от вас термин из двух слов, обозначающий современную область информатики. Напишите этот термин. Ответ: Машинное обучение. Зачет: Machine learning. Комментарий: Мониторы на картинке расставлены так, как будто они внимают речам более широкого телевизора. Источник: 1. (pic: 20161021.jpg) 2. https://ru.wikipedia.org/wiki/Машинное_обучение Автор: Даниил Шункевич (Минск) Вопрос 29: Один из комментаторов на сайте habrahabr.ru [хабрахАбр точка ру] шутит, что основная проблема с НИМ - это понять, работает он или нет. Назовите предмет шутки двумя словами, начинающимися на одну и ту же букву. Ответ: Квантовый компьютер. Комментарий: Состояние квантового компьютера, как и кота Шрёдингера, описывается квантовой суперпозицией. Если кот Шрёдингера одновременно жив и мертв, то, возможно, квантовый компьютер одновременно работает и не работает. Источник: https://geektimes.ru/post/267324/#comment_8907406 Автор: Андрей Танана (Минск) Вопрос 30: Согласно книге "Одноэтажная Америка", чрезвычайно практичные американцы повесили возле НЕЕ табличку, сообщающую, что из НЕЕ можно было бы построить 40 домов по пять комнат в каждом. Назовите ЕЕ максимально точно. Ответ: [Секвойя] "Генерал Шерман". Комментарий: Секвойя "Генерал Шерман" - первое по объему и массе дерево на Земле. Высота "Генерала Шермана" - 83,8 метра, масса оценивается в 1900 тонн, а объем ствола - в 1487 кубических метров. Источник: И. Ильф, Е. Петров. Одноэтажная Америка. http://flibusta.is/b/168602/read Автор: Вадим Кузмич (Логойск) Вопрос 31: Влад Цепеш является прототипом графа Дракулы. Статья о Цепеше и его потомках, опубликованная в журнале "Вокруг света", называется "НАСТОЯЩИЕ ДЕТЕКТИВЫ". Название какого фильма мы заменили? Ответ: "Реальные упыри". Комментарий: В отличие от Дракулы, Цепеш существовал в реальности. Слова "настоящие" и "реальные" - синонимы. Источник: "Вокруг света", 2016, N 8, обложка. Автор: Евгений Лешкович (Минск) Вопрос 32: (pic: 20161022.jpg) В последней строке раздаточного материала упоминается изобретение Альфреда Баттса, придуманное им в 1938 году. Назовите это изобретение. Ответ: Scrabble. Зачет: Скрэббл; Эрудит. Комментарий: Последняя строка: "Сыграть партию в Скрэббл и не получить по лицу". Источник: 1. https://xkcd.com/1750/ 2. https://en.wikipedia.org/wiki/Scrabble Автор: Виталий Калачёв (Минск) Вопрос 33: В августе 1893 года известный человек остановился в отеле "Du Sauvage" [дю совАж]. Прогуливаясь по окрестностям с друзьями, этот человек посетовал: "Он отвлекает мои мысли от более важных дел", после чего человека отвели к ИКСУ. Назовите ИКС двумя словами. Ответ: Рейхенбахский водопад. Комментарий: Артур Конан Дойл жаловался друзьям, что не знает, как избавиться от Шерлока Холмса. Сэр Генри Ланн посоветовал писателю столкнуть его в Рейхенбахский водопад, который затем ему и показал вживую. Источник: http://www.vokrugsveta.ru/article/259112/ Автор: Павел Новиков (Витебск - Минск) Вопрос 34: Недавно к НИМ официально присоединился пингвин, появившийся благодаря обнаруженной в архивах детской фотографии. Назовите ИХ, использовав два дефиса. Ответ: Все-все-все. Зачет: "Винни-Пух и все-все-все". Комментарий: ОНИ - друзья Винни-Пуха. В названии перевода Бориса Заходера - те самые "все-все-все". К 90-летию персонажа выпустили новый официальный сборник рассказов. Источник: https://www.theguardian.com/books/2016/sep/19/winnie-the-pooh-penguin-anniversary-aa-milne Автор: Виталий Калачёв (Минск) Вопрос 35: Уроженец Ханьчжоу Шень Ко был выдающимся математиком, астрологом, этнографом, поэтом и дипломатом. Рассказывая о Шень Ко, Эрик Вейнер называет его ТАКИМ ИМ. Ответьте, используя два слова, начинающиеся на соседние буквы: что мы заменили словами "ТАКОЙ ОН"? Ответ: Китайский Леонардо. Комментарий: Шень Ко был настоящим человеком Возрождения. Правда, родился он за триста лет до Возрождения. Источник: Э. Вейнер. География гениальности. Где и почему рождаются великие идеи. http://flibusta.is/b/465171/read Автор: Евгений Лешкович (Минск) Вопрос 36: Владелец футбольного клуба "Уимблдон" Сэм Хаммам прописывал в контракте каждого игрока, что в случае победы с разницей в четыре мяча тот пойдет в оперу. Какое слово в предыдущем предложении мы заменили? Ответ: Поражения. Комментарий: Хаммам неплохо знал своих футболистов и хорошо понимал, что высидеть оперу для них будет не наградой, а наказанием. Источник: И. Калашников. Мир английского футбола. Знаменитые клубы, легендарные игроки и драматичные сюжеты. https://books.google.ru/books?id=kcI_DQAAQBAJ&pg=PT81#v=onepage&q&f=false Автор: Евгений Лешкович (Минск) Тур: 3 этап Дата: 10-Dec-2016 Редактор: 1-18 - Аркадий Рух (Минск); 19-36 - Павел Свердлов (Минск) Инфо: Аркадий Рух благодарит за тестирование и ценные замечания: Алексея Морозова (Тула), Ирину Зубкову (Нижний Новгород), Андрея Кокуленко (Омск), Виталия Пронькина (Реутов), Андрея Данченко (Винница), Александра Немировского (Киев), Серафима Шибанова (Москва), Дмитрия Когана (Штутгарт), Александра Кудрявцева (Николаев), Бориса Моносова (Санкт-Петербург), Яну Азриэль (Хайфа), Николая Коврижных (Москва), Артема Корсуна (Харьков), Анну Виниченко (Минск). Особая благодарность Ивану Топчию (Минск). Вопрос 1: В анекдоте ТАКИЕ парашютисты полностью деморализовали армию противника. ТАКОЕ королевство существовало с 854 по 1707 год. Какое - ТАКОЕ? Ответ: Шотландское. Комментарий: В 1707 году Шотландское и Английское королевства были объединены в Великобританию. Килт - традиционная шотландская мужская юбка - носится без нижнего белья. Источник: 1. https://www.anekdot.ru/id/634868/ 2. https://ru.wikipedia.org/wiki/Шотландское_королевство Автор: Аркадий Рух (Минск) Вопрос 2: В статье, опубликованной летом 2016 года, американский журналист Карен Зраик пишет, что "многие уже попали на первые полосы газет из-за сомнительного решения ДЕЛАТЬ ЭТО в Освенциме, на Арлингтонском кладбище и на мемориале 11 сентября". Ответьте двумя словами: что делать? Ответ: Ловить покемонов. Комментарий: Речь в статье идет о сверхпопулярной игре "Покемон Го", увлечение которой стало настоящей эпидемией. Покемонов, в самом деле, порою ловят в самых неподходящих для этого местах. Источник: https://www.inopressa.ru/article/21Jul2016/nytimes/pokem.html Автор: Аркадий Рух (Минск) Вопрос 3: В фильме "Маленькие пальчики" Гэри Олдмен и Питер Динклейдж сыграли членов одной семьи. На съемках Олдмену приходилось всё время УНИЖАТЬСЯ. Какие три слова мы заменили словом "УНИЖАТЬСЯ"? Ответ: Стоять на коленях. Комментарий: Фильм рассказывает о семье лилипутов. Невысокий рост Питера Динклейджа хорошо известен, например, по роли Тириона Ланнистера в телесериале "Игра Престолов". Гэри Олдмен - актер нормального роста, тем не менее, приглашенный играть брата главного героя. Поэтому его никогда не показывают в полный рост, а чтобы актер казался ниже, он постоянно находился на коленях. Источник: https://www.youtube.com/watch?v=S36wab0hrNE Автор: Аркадий Рух (Минск) Вопрос 4: Персонаж цикла романов "Песнь Льда и Огня" рыцарь ПАтрек с Королевской Горы получил свое имя в честь ПатрИка Сен-ДенИ, друга Джорджа Мартина. Назовите родной город ПатрИка Сен-ДенИ. Ответ: Монреаль. Комментарий: Столица Квебека - франкоговорящей провинции Канады. Собственно, название города и переводится как "Королевская гора". Источник: http://7kingdoms.ru/wiki/Патрек_с_Королевской_Горы Автор: Аркадий Рух (Минск) Вопрос 5: Джордж Мартин пишет, что ЭТО короткое СЛОВО часто сопровождается разочарованиями и огорчениями. Напишите ЭТО СЛОВО, которое в оригинале в два раза короче, чем в русском переводе, и часто используется в качестве примера лаконичности. Ответ: Если. Зачет: If. Комментарий: "Если" - это еще и знаменитый ответ спартанцев Филиппу Македонскому на угрозу уничтожить город, если он его захватит. Источник: 1. Дж. Мартин. Чумная звезда. https://books.google.ru/books?id=UfmmAAAAQBAJ&pg=PA5-IA9#v=onepage&q&f=false 2. https://ru.wikipedia.org/wiki/Лаконичность Автор: Аркадий Рух (Минск) Вопрос 6: В песенке команды КВН "Рижские готы" говорится о том, что одна женщина настолько плохо справляется со своими обязанностями, что ей не страшен даже электрический стул. Назовите ее профессию. Ответ: Проводник. Зачет: Проводница. Комментарий: Проводнице по имени Даша Электрический стул не так страшен. Хоть разряд был велик, В Дашу он не проник: Даша очень плохой проводник! Источник: КВН-2012. Сборная Прибалтики. 1/8 финала. Приветствие. https://www.youtube.com/watch?v=D0SnueSqxrU&t=3m07s Автор: Аркадий Рух (Минск) Вопрос 7: В январе 1980 года в Белграде были внимательно изучены видеозаписи Уинстона Черчилля, Джона Кеннеди, графа Маунтбеттена и других. В ноябре 1982 году в СССР без подобных записей смогли обойтись. Какое слово мы пропустили в вопросе? Ответ: Похорон. Комментарий: За долгие годы правления Иосипа Броза Тито в Югославии была напрочь утрачена традиция похорон первых лиц государства. Пришлось обращаться к "зарубежному опыту". В 1982 году, когда в СССР умер Брежнев, таких проблем не возникло. Источник: https://ru.wikipedia.org/wiki/Смерть_и_похороны_Иосипа_Броза_Тито Автор: Аркадий Рух (Минск) Вопрос 8: Владелец одного британского бара заметил, что даже самые сплоченные семьи и дружные компании в его заведении превращаются в сборище одиночек. Чтобы вернуть старые добрые времена, владелец воспользовался изобретением, сделанным 180 лет назад. Ответьте двумя словами, одно из которых - имя собственное: как называется это изобретение? Ответ: Клетка Фарадея. Зачет: Комната Фарадея; щит Фарадея. Комментарий: Клетка Фарадея, экранирующая электромагнитное поле, не позволяет посетителям бара пользоваться мобильной связью, wi-fi и прочими "достижениями цивилизации", вынуждая людей общаться между собой. Источник: 1. https://www.telegraph.co.uk/science/2016/08/02/landlord-installs-faraday-cage-to-block-phone-signals-because-so/ 2. https://ru.wikipedia.org/wiki/Клетка_Фарадея Автор: Аркадий Рух (Минск) Вопрос 9: [Ведущему: кавычки не озвучивать.] Знакомая автора вопроса жалуется, что ее ребенок-подросток слишком увлечен "Вселенной Стивена" и вместо книг читает только ИХ. Назовите ИХ словом французского происхождения. Ответ: Субтитры. Незачет: Титры. Комментарий: Девочка увлечена мультсериалом "Вселенная Стивена" и читает преимущественно субтитры к нему. Титры, показываемые перед и после серии, ее не особо интересуют. Источник: 1. http://memnoga.livejournal.com/26748.html 2. https://ru.wikipedia.org/wiki/Субтитры 3. https://ru.wikipedia.org/wiki/Титры Автор: Аркадий Рух (Минск) Вопрос 10: Внимание, в вопросе есть замены. Статья на сайте Газета.ру, посвященная кризису экономики одной из нефтедобывающих стран, называется "ПЕРВЫЕ становятся ВТОРЫМИ". В экранизации 1972 года Витя Смирнов сыграл и ПЕРВОГО, и ВТОРОГО. Какие слова мы заменили словами "ПЕРВЫЙ" и "ВТОРОЙ"? Ответ: Принц, нищий. Комментарий: Статья "Принцы становятся нищими" посвящена экономическому кризису в Саудовской Аравии, вызванному падением цен на нефть. "Принц и нищий" - неоднократно экранизированный роман Марка Твена о том, как маленький попрошайка Том Кенти внезапно оказался двойником наследника английского престола. Источник: 1. https://www.gazeta.ru/business/2016/09/29/10221485.shtml 2. https://ru.wikipedia.org/wiki/Принц_и_нищий_(фильм,_1972) Автор: Аркадий Рух (Минск) Вопрос 11: [Ведущему: в словах "АЛЬФЕ" и "АЛЬФОМ" как можно точнее проговорить окончания.] Внимание, в вопросе есть замены. В материале, посвященном Всемирному дню почты, говорится, что на первой "АЛЬФЕ с АЛЬФОМ" были изображены "Обрученные и Эйфелева башня". А вот на белорусской "АЛЬФЕ с АЛЬФОМ", появившейся в 1993 году, изображена "Прогулка". Какое слово мы заменили словом "АЛЬФА"? Ответ: Марка. Комментарий: Речь идет о "марках с Марком" - филателистической продукции, посвященной творчеству великого белорусского художника Марка Шагала. Источник: http://fun.rambler.ru/special/vsemirnyj-den-pochty/marka-s-markom/ Автор: Аркадий Рух (Минск) Вопрос 12: Французское слово "шифр" происходит от арабского "сифр", обозначающего ЕГО. В латинских переводах XII века для ЕГО обозначения использовалось слово "циркулюс". Назовите ЕГО. Ответ: Ноль. Комментарий: Ту же этимологию имеет и слово "цифра". Циркулюс, или "кружок", - вполне подходящее название для ноля. Источник: 1. https://ru.wikipedia.org/wiki/Шифр 2. https://ru.wikipedia.org/wiki/Ноль_(цифра) Автор: Аркадий Рух (Минск) Вопрос 13: В произведении Рэя Брэдбери ОН сначала "смаковал картины Пикассо и Матисса", а затем перекрашивал портьеры в спальне. По одной из версий, слово "ОН" происходит от имени древнеиндийского божества. Назовите ЕГО. Ответ: Огонь. Незачет: Пожар. Комментарий: Брэдбери описывает пожар. Русское слово "огонь" родственно санскритскому Агни - богу огня. Источник: 1. Р. Брэдбери. Будет ласковый дождь. http://flibusta.is/b/181956/read 2. https://ru.wikipedia.org/wiki/Огонь 3. https://ru.wikipedia.org/wiki/Агни Автор: Аркадий Рух (Минск) Вопрос 14: В 1921 году некий Бартоломеус Шпакович застрелил последнего беловежского зубра. Зоолог Эрна Морн сравнила Шпаковича с НИМ, хотя руководствовалась причинами, прямо противоположными тем, благодаря которым ОН попал в историю. Назовите ЕГО. Ответ: Герострат. Комментарий: Эрна Морн считает, что имя Шпаковича, как и имя Герострата, должно жить в веках, как предостережение. Самого Герострата, напротив, было приказано забыть. Источник: https://life.ru/t/наука/903075/zdies_byli_lvy_kratkaia_istoriia_brakonierstva Автор: Аркадий Рух (Минск) Вопрос 15: В рассказе Евгения Лукина орел, терзавший Прометея, носит имя арабского происхождения. Назовите это имя. Ответ: Алкоголь. Комментарий: Орел, как известно, клевал Прометею печень. А "цирроз" - слово греческого происхождения. Источник: 1. Е.Ю. Лукин. Произведение искусства. http://flibusta.is/b/327116/read#t35 2. https://ru.wikipedia.org/wiki/Алкоголь Автор: Аркадий Рух (Минск) Вопрос 16: Герой новеллы Теодора Гофмана, впервые опубликованной во "Всеобщей музыкальной газете", рассказывает о том, как в 1809 году герой встретил ЕГО спустя двадцать лет после ЕГО смерти. Назовите ЕГО односложную фамилию. Ответ: Глюк. Комментарий: По сюжету великий австрийский композитор Кристофор Виллибальд Глюк был так возмущен берлинской постановкой своей оперы, что решил вмешаться. "Глюк" - это еще и разговорное название галлюцинации. Источник: 1. https://ru.wikipedia.org/wiki/Кавалер_Глюк 2. https://ru.wikipedia.org/wiki/Глюк,_Кристоф_Виллибальд 3. https://ru.wikipedia.org/wiki/Глюк_(значения) Автор: Аркадий Рух (Минск) Вопрос 17: Британская "Дейли мейл" отмечает роль феминизма в том, что современные женщины стали заботиться о своем ИКСЕ. Можно сказать, что без изобретения другого ИКСА никакой "Дейли мейл" бы не было. Какое слово мы заменили ИКСОМ? Ответ: Пресс. Комментарий: Газета пишет, что с ростом влияния феминизма женщины стали больше следить за своей физической формой, в частности - за состоянием мышц брюшного пресса. Без изобретения печатного пресса появление "Дейли мейл" было бы невозможно. Источник: 1. https://lenta.ru/articles/2015/10/21/waist/ 2. http://finances.social/biznes_760_762/pechatnyiy-press-39703.html Автор: Аркадий Рух (Минск) Вопрос 18: Один из элементов награды, вручавшейся в Советском Союзе с 1932 по 1941 год, состоял из нескольких концентрических кругов. Мы не спрашиваем, какое имя собственное входило в название награды. Ответьте, за достижения в какой области она вручалась. Ответ: Стрельба. Комментарий: В нижней части значка "Ворошиловский стрелок", вручаемой за меткость, находилось изображение круглой мишени. Редактор тура надеется, что сегодня команды поразили все мишени. Источник: https://ru.wikipedia.org/wiki/Ворошиловский_стрелок Автор: Аркадий Рух (Минск) Вопрос 19: <раздатка> Победитель - тот, кто солнце на один раз больше, чем давление. Какие два глагола мы поменяли на существительные в цитате из книги Мии Хэмм? Ответ: Встает, падает. Зачет: Поднимается, падает и т.п. по смыслу. Комментарий: Солнце тоже встает, а давление тоже падает. Источник: https://www.goodreads.com/quotes/324217-a-winner-is-that-person-who-gets-up-one-more Автор: Ольга Потапова ("Семь сорок") Вопрос 20: Тим Бернерс-Ли как-то сказал, что если бы он в свое время мог предположить, сколько людей будет пользоваться Интернетом, то ни за что не стал бы использовать... Что? Изобразите ответ на своих карточках. Ответ: // Зачет: :// Незачет: www, http и т.д. Комментарий: Изобретатель Интернета сделал бы синтаксис проще, в частности, отказавшись от двух слэшей в URL. Источник: https://citaty.info/quote/364474 Автор: Павел Свердлов ("Семь сорок") Вопрос 21: Жесткая конкуренция со "Standard Oil" привела к слиянию двух компаний, одна из которых в год своего основания заручилась поддержкой короля Виллема III. Назовите то, что получилось после слияния. Ответ: "Royal Dutch Shell". Зачет: "Shell". Комментарий: Поддержка короля Нидерландов Виллема III отразилась на названии компании "Royal Dutch". В 1907 году RD объединилась с "The Shell Transport and Trading Company". Новая компания стала называться "Royal Dutch Shell". Или просто "Shell". Источник: http://www.fundinguniverse.com/company-histories/royal-dutch-petroleum-company-the-shell-transport-and-trading-company-p-l-c-history/ Автор: Павел Свердлов ("Семь сорок") Вопрос 22: В 1950-х годах ученые, в лабораторных условиях исследовавшие поведение, смогли управлять АЛЬФОЙ как ИГРЕКОМ. Что мы заменили АЛЬФОЙ и ИГРЕКОМ? Ответ: Мышь, джойстик. Зачет: Мышь, курсор. Комментарий: В лабораториях бывают мыши. Бывает, их там бьют током. Бывает, что после этого они ходят так, как этого хотят ученые. Мышь и джойстик - два компьютерных девайса. Источник: https://psyfactor.org/lib/dofamin.htm Автор: Александра Бурчалова ("Семь сорок") Вопрос 23: Очевидцы сообщали, что однажды в начале 1918 года на Кузнецком мосту в Москве из-за НЕЕ образовался небольшой затор. С каким глаголом в том же году срифмовал ЕЕ известный поэт? Ответ: Клёшить. Комментарий: Уставшую лошадь, упавшую на Кузнецком мосту, Маяковский сделал героем стихотворения "Хорошее отношение к лошадям". В нем есть строки: "Лошадь на круп грохнулась, и сразу за зевакой зевака, штаны пришедшие Кузнецким клёшить, сгрудились, смех зазвенел и зазвякал: - Лошадь упала! - Упала лошадь!". Источник: 1. http://www.itaka.msk.ru/archives/617 2. В.В. Маяковский. Хорошее отношение к лошадям. http://www.feb-web.ru/feb/mayakovsky/texts/ms0/ms2/ms2-010-.htm Автор: Павел Свердлов ("Семь сорок") Вопрос 24: В вопросе есть замена. Николай Кононов пишет, что название "ИКС" пришло в голову сразу. Изначально предполагалось добавлять к "ИКСУ" городА (например, "ИКС с Хабаровском"), но от этой идеи быстро отказались. Также герой книги рассуждает о пользе заимствования технологий из других стран. Что мы заменили ИКСОМ? Ответ: "ВКонтакте". Комментарий: Речь в вопросе идет про книгу Николая Кононова "Код Дурова", посвященную истории создания соцсети. В одном из отрывков книги приводится точка зрения Дурова на обвинения в заимствовании идеи и функционала соцсети у американской Facebook. Источник: http://www.forbes.ru/sobytiya-opinion/lyudi/212150-kod-pavla-durova-pyat-istorii-iz-zhizni-vkontakte-i-ee-sozdatelya Автор: Станислав Габрусевич ("Семь сорок") Вопрос 25: Согласно анонимной цитате из Интернета, если ваша АЛЬФА толстая, то вам не хватает физических нагрузок. Назовите композитора, с которым с 1992 года ассоциируется одна АЛЬФА. Ответ: Бетховен. Комментарий: АЛЬФА - это собака. Бетховен - это не только композитор, но и кличка собаки из одноименного фильма. Источник: 1. https://www.dogquotations.com/dog-quotes-author-unknown2.html 2. https://en.wikipedia.org/wiki/Beethoven_(film) Автор: Алексей Евдоченко ("Семь сорок") Вопрос 26: Советский крейсер, построенный в 1937 году, назвали в честь Вячеслава Молотова. Через двадцать лет, после разгрома "антипартийной группы" Молотова, Кагановича и Маленкова, корабль переименовали. Новое название выбрали то ли с издевкой, то ли чтобы на всякий случай не обидеть опального министра. Какое название стал носить крейсер? Ответ: "Слава". Источник: https://mi3ch.livejournal.com/3569303.html Автор: Егор Сидорович ("Семь сорок") Вопрос 27: (pic: 20161023.jpg) Андрей Громыко больше трех десятков лет находился на вершине советской партийной номенклатуры. Описывая стиль группы "Громыка", журналист Кирилл Ступников добавляет одну букву к понятию советского времени. Назовите получившийся в результате неологизм. Ответ: Политбюрок. Источник: http://www.km.ru/muzyka/2016/03/18/persony-i-sobytiya-v-mire-muzyki/773707-gromyka-gromyka Автор: Егор Сидорович ("Семь сорок") Вопрос 28: ЕГО старшего брата, который погиб на фронтах Второй мировой войны, звали Маркс. В 2007 году ОН, в числе других ученых, подписал открытое письмо Владимиру Путину против клерикализации общества. Назовите ЕГО. Ответ: [Жорес] Алфёров. Комментарий: Будущего физика родители тоже назвали в честь известного коммуниста - Жана Жореса. Алфёров - физик, поэтому его борьба с клерикализмом выглядит вполне органично. Источник: https://ru.wikipedia.org/wiki/Алфёров,_Жорес_Иванович Автор: Павел Свердлов ("Семь сорок") Вопрос 29: Российский антифашист Петр Силаев вспоминал, как в школе тайком прочитал одну книгу, распечатав ее на принтере. Помимо драк в барах и суровых военных сцен, внимание в книге уделялось подавлению одной советской республики. По мнению Силаева, это событие сыграло едва ли не поворотную роль в мировоззрении автора. В каком городе располагалась столица этой республики? Ответ: В Мюнхене. Комментарий: В вопросе речь идет о книге "Майн Кампф", которую Силаев, желая "изучить врага", тайно распечатал в школьном кабинете информатики. В книге Гитлер уделял внимание разгрому в 1919 году Баварской советской Республики, чье краткое существование стало для будущего лидера нацистов доказательством реальности советско-еврейской угрозы. Источник: https://daily.afisha.ru/archive/vozduh/books/8-knig-o-separatizme/ Автор: Станислав Габрусевич ("Семь сорок") Вопрос 30: ТАКОЙ ОН, изображенный на рекламе мобильного оператора, вызывает ассоциации с двумя крупными конкурирующими сегментами рынка гаджетов. Из рекламы следует, что первый взнос за смартфон составит ноль рублей. Что мы заменили словами "ТАКОЙ ОН"? Ответ: Надкусанный пончик. Зачет: Надкусанный донат. Комментарий: Ассоциации сразу и с логотипом "Apple", и с ОС Android (Android 1.6 назывался "Donut", и вообще все версии Android называются в честь чего-нибудь сладенького). Источник: Реклама velcom в Минском метро. Автор: Павел Свердлов ("Семь сорок") Вопрос 31: В вопросе есть замена. Журнал "Arzamas" рассказывает о том, как торговцы из одной страны заметили, что рыба лучше сохраняется, если пересыпать ее ИКСОМ. Поначалу ИКС выбрасывали, как использованную упаковку, но потом нашли лучшее применение. Мы не спрашиваем, что мы заменили ИКСОМ. Ответьте, в какой стране это происходило. Ответ: Япония. Комментарий: Торговцы из японского портового города Осака возили в удаленные места рыбу, для большей сохранности пересыпанную ферментированным рисом. Со временем рыбно-рисовую смесь стали есть, что впоследствии трансформировалось в суши. Источник: https://arzamas.academy/materials/745 Автор: Станислав Габрусевич ("Семь сорок") Вопрос 32: Психолог Гелена Савицкая проводит аналогии между людьми и архитектурными элементами. С чем она сравнивает женщин, которые много на себя берут и пытаются всё контролировать, но реальной пользы почти не приносят? Ответ: С кариатидой. Комментарий: В реальном мире кариатида пользы приносит куда меньше, чем можно предположить по виду. Источник: https://gelena-s.livejournal.com/440983.html Автор: Ольга Потапова ("Семь сорок") Вопрос 33: Спортивный журналист Кирилл Новокщёнов сравнил ИКС хоккеиста Наиля Якупова со среднегодовой температурой на Аляске. Назовите ИКС двумя словами, начинающимися на одну и ту же букву. Ответ: Показатель полезности. Зачет: Показатель плюс-минус. Комментарий: Среднегодовая температура на Аляске - так себе. Источник: https://www.sports.ru/tribuna/blogs/thethinblueline/1075534.html Автор: Алексей Евдоченко ("Семь сорок") Вопрос 34: Сотрудница Третьяковской галереи Ирина Вакар рассказывает, что ОН предлагал разные интерпретации своего произведения, в том числе и в иронической манере. Этим можно объяснить наличие под слоем краски надписи, отсылающей к картине французского художника. Кто фигурирует в названии этой картины? Ответ: Негры. Комментарий: В конце 2015 года сотрудники Третьяковской галереи, проводившие исследования "Черного квадрата" Малевича, обнаружили, что то, что считалось ранее подписью художника, является надписью "битва негров ночью". Надпись отсылает к картине эксцентричного французского художника Альфонса Алле "Битва негров в пещере глубокой ночью". Долгое время связь "Черного квадрата" и "Битвы негров..." расценивалась как исторический анекдот. Теперь же стало известно, что Малевич как минимум знал про картину Алле. Источник: https://meduza.io/feature/2015/11/19/kazimir-malevich-v-peschere-glubokoy-nochyu Автор: Станислав Габрусевич ("Семь сорок") Вопрос 35: Сравнивая бизнес-тренинги, необходимые для того, чтобы не отстать от конкурентов, с тренировкой на беговой дорожке, Райля Фофанова упоминает реплику из книги, изданной в 1871 году. Назовите персонажа, который произносит эту реплику. Ответ: Черная королева. Комментарий: Она говорила, что "приходится бежать со всех ног, чтобы только остаться на том же месте". Совсем как на беговой дорожке. А у нас в пакете два вопроса подряд про черный цвет. Источник: 1. "Бизнес-журнал", 2007/18: Архангельская область. https://books.google.ru/books?id=_hgOCgAAQBAJ&pg=PA16#v=onepage&q&f=false 2. https://en.wikipedia.org/wiki/Through_the_Looking-Glass Автор: Александра Бурчалова, Павел Свердлов ("Семь сорок") Вопрос 36: Согласно одному толкованию, долгожданное событие произойдет не раньше, чем все евреи вернутся в Израиль. Поэтому львовянин Павел Машиах шутит, что переедет на ПМЖ в Землю Обетованную... Каким? Ответ: Последним. Комментарий: Событие, о котором идет речь, - приход Мессии. Фамилия "Машиах" переводится с иврита как "Мессия", да и звучит похоже. Павел собирается переехать в Израиль с пафосом, а у нас этот вопрос, как и ответ, был последним. Источник: 1. http://www.moshiach.ru/moshiach/KingMoshiach/28.html 2. Беседа автора вопроса с Павлом Машиахом. Автор: Павел Свердлов ("Семь сорок") Тур: 4 этап Дата: 25-Feb-2017 Редактор: Иван Топчий и Павел Малецкий (Минск) Инфо: Ведущие команды: "Хронически Разумные United", "Автостопом по Прибалтике", "Не вопрос", "Zемляне", "Ять". Редакторы благодарят за помощь и ценные советы: Валерия Семёнова, Дарью Соловей, Марию Петровскую, команды "Садик Хана Дзавы", "Конструктор" и "Кто все эти люди?". Вопрос 1: [Ведущему: четко прочитать "и" в слове "Кастилтэка".] ПЕРВЫЕ назвали ВТОРЫХ - КастилтЭка. Назовите и ПЕРВЫХ, и ВТОРЫХ. Ответ: Ацтеки, испанцы. Зачет: Ацетки, кастильцы/конкистадоры/конквистадоры. Незачет: Индейцы вместо ацтеков. Комментарий: Испанцы всё еще называли свою страну Кастилией в честь центрального королевства. Источник: https://www.reddit.com/r/AskHistorians/comments/5dtdbx/did_the_aztecs_understand_where_the_spanish_were/da7wb0t/ Автор: Алексей Волчок ("Хронически разумные United") Вопрос 2: <раздатка> "__________, My Déjà Vu" Перед вами название одного из эпизодов телесериала "Клиника". Напишите три слова, которые мы пропустили в этом названии. Ответ: My Déjà Vu. Зачет: По словам "Déjà Vu". Комментарий: Серия называется "My Déjà Vu, My Déjà Vu". Источник: https://en.wikipedia.org/wiki/List_of_Scrubs_episodes Автор: Павел Малецкий ("Хронически разумные United") Вопрос 3: В средневековой Англии служители церкви имели право избежать светского суда и тем самым смертной казни. Начиная с XV века под действие этой привилегии стали попадать многие аристократы, а иногда даже люди низших сословий. Положение было отменено только в 1827 году, когда ДЕЛАТЬ ЭТО могла уже бОльшая часть населения. Что именно делать? Ответ: Читать. Зачет: Писать. Комментарий: До появления книгопечатания читать умело только духовенство, поэтому считалось, что все, кто умеют читать - духовенство. Источник: А.Ф. Кистяковский. Исследования о смертной казни. - М: Директ-Медиа, 2010. - С. 148. Автор: Елизавета Лысенко ("Не вопрос") Вопрос 4: [Ведущему: обязательно прочитать комментарий!] В нижней части картины Василия Балабанова "Пловец" можно увидеть как бы отраженное в воде известное архитектурное сооружение. Говоря о своей картине, Василий Балабанов упомянул Второе пришествие Христа. Какие два слова мы пропустили в предыдущем предложении? Ответ: Храм, Спасителя. Комментарий: По замыслу автора Храм Христа Спасителя как бы отражается в водах бассейна "Москва", который изображен в верхней части картины. Источник: https://ru.wikipedia.org/wiki/Пловец_(картина) Автор: Павел Малецкий, Герман Чепиков ("Хронически разумные United") Вопрос 5: <раздатка> Дело объяснялось очень просто: "король" - было только такое слово, и больше ничего. Никакой власти королю не давалось. Всякий делал, что хотел, и это называлось DURA LEX... И частенько по ночам плакали короли в подушку от этого DURA LEX... Перед вами цитата из пародийной "Всеобщей истории, обработанной "Сатириконом"". Что мы заменили словами "dura lex" [дУра лекс]? Ответ: Liberum veto. Зачет: Либерум вето. Комментарий: Система права Речи Посполитой, позволявшая любому депутату Сейма наложить вето на общее решение. Королевская власть в Речи Посполитой была крайне слаба. Источник: Всеобщая история, обработанная "Сатириконом". http://az.lib.ru/a/awerchenko_a_t/text_0120.shtml Автор: Иван Топчий ("Хронически разумные United") Вопрос 6: Британец Алекс Янгер в интервью заявил, что благодарен ЕМУ за популярность и разделяет ЕГО любовь к техническим новинкам. Тем не менее, Янгер признаётся, что никогда не нанял бы ЕГО на работу из-за опрометчивости и аморальности. Мы не просим вас назвать ЕГО. Назовите организацию, которую возглавляет Янгер. Ответ: МИ-6. Зачет: MI6; Секретная разведывательная служба; Secret Intelligence Service; Военная разведка; Military Intelligence. Комментарий: Таким образом глава британской спецслужбы отозвался о Джеймсе Бонде. Источник: https://www.theguardian.com/uk-news/2016/dec/08/james-bond-would-not-get-job-with-real-mi6-says-spy-chief Автор: Алексей Волчок ("Хронически разумные United") Вопрос 7: Современники отмечали, что в ЕГО квартире повсюду можно было увидеть чертежи, комки глины, грязь и даже навоз. Назовите ЕГО фамилию. Ответ: Клодт. Комментарий: Своих знаменитых лошадей он изображал с натуры, для чего приводил их в свою комнату. Источник: http://www.cablook.com/design-art/lyubimye-loshadi-barona-klodta/ Автор: Сергей Поляков ("Summa Technologiae"), в редакции Павла Малецкого ("Хронически разумные United") Вопрос 8: Описывая поездку Пушкина по Заволжью в 1833 году, современный критик Алексей Иванов так отозвался о слове, впервые услышанном поэтом: "Слово, пойманное в степи, Пушкин добросил до орбиты Земли". Назовите это слово. Ответ: Буран. Комментарий: В поездке Пушкин собирал материал о пугачёвском восстании и дивное слово запомнил: "Ну барин", - закричал ямщик, - "беда: буран!" ("Капитанская дочка"). А Иванов намекает на аэрокосмический челнок "Буран". Источник: А.В. Иванов. Вилы. https://books.google.ru/books?id=TTlCDQAAQBAJ&pg=PT19#v=onepage&q&f=false Автор: Надежда Лейчинская ("Ять") Вопрос 9: В юности баскетболист Джеральд Грин тренировал броски сверху, используя некачественные самодельные баскетбольные кольца. В 2007 году Грин стал первым в истории ТАКИМ победителем конкурса слэм-данков на матче всех звезд НБА. Назовите любой сериал, большинство героев которого - ТАКИЕ. Ответ: "Симпсоны". Зачет: "Футурама"; "Гриффины". Комментарий: ТАКИЕ - это четырехпалые. Джеральд Грин потерял один из пальцев на руке, когда забил сверху в самодельное кольцо, из которого торчал шуруп. У большинства персонажей "Симпсонов" (кроме Бога и Иисуса) на всех конечностях по четыре пальца. Источник: 1. https://shkola2-0.ru/ucheniku/lifestyle/bullying 2. https://en.wikipedia.org/wiki/List_of_recurring_The_Simpsons_characters Автор: Валентин Копочель ("Автостопом по Прибалтике") Вопрос 10: Светлокожий баскетболист Джейсон Уильямс за уличный стиль игры и частые нарушения дисциплины получил в НБА прозвище ИКС. Впервые ИКС был выпущен в 1930-х годах в Швейцарии. Какие два слова мы заменили ИКСОМ? Ответ: Белый шоколад. Комментарий: Джейсон Уильямс является белокожим, что не мешало ему своим поведением походить на типичного афроамериканца. Источник: 1. https://www.sports.ru/tribuna/blogs/spursmania/861325.html 2. https://ru.wikipedia.org/wiki/Белый_шоколад Автор: Валентин Копочель ("Автостопом по Прибалтике") Вопрос 11: [Ведущему: обязательно прочитать комментарий!] Рассказывая о послевоенных годах, Иван Лихач приводит в пример человека по фамилии Мычко, который знал пять ИХ, а работал председателем сельсовета. Назовите любую из этих пяти ИХ. Ответ: М. Зачет: ы; ч; к; о. Комментарий: Мычко знал только, как писать собственную фамилию. Сам Лихач закончил четыре класса школы и считался человеком грамотным. Источник: https://news.tut.by/society/488638.html Автор: Алексей Гончаров ("Хронически разумные United") Вопрос 12: [Ведущему: обязательно прочитать комментарий!] Прибывший после войны в родную деревню Иван Лихач был назначен ЕЕ руководителем. Поскольку на тот момент ОНА была пустой комнатой, можно сказать, что ОНА дважды не соответствовала своему названию. Назовите ЕЕ словом, пишущимся через дефис. Ответ: Изба-читальня. Комментарий: Как вы уже знаете, Иван Лихач слыл человеком грамотным. В комнате не было книг и газет, поэтому сложно было назвать ее как избой, так и читальней. Заканчивая историю про Ивана Лихача, можно сказать, что впоследствии он сделал удачную карьеру, в конце которой стал первым секретарем Центризбиркома независимой Беларуси. Источник: https://news.tut.by/society/488638.html Автор: Алексей Гончаров ("Хронически разумные United") Вопрос 13: (pic: 20161024.jpg) Будущий вице-президент США Элбридж Герри применил методику, называемую теперь его именем. На картинке выделен один из НИХ в Чикаго. Назовите ЕГО двумя словами. Ответ: Избирательный округ. Зачет: Мажоритарный округ. Незачет: Избирательный участок. Комментарий: Герри применил практику джерримандеринга - перераспределения территорий избирательных округов с целью манипуляции результатами выборов по мажоритарной системе. Так, 100 человек, из которых 70 белых и 30 черных, можно объединить в 10 равных групп так, что черные будут большинством в половине групп либо ни в одной. На рисунке один-единственный избирательный округ накрывает большинство районов Чикаго, где живут латиноамериканцы. Источник: 1. https://en.wikipedia.org/wiki/Illinois's_4th_congressional_district 2. https://ru.wikipedia.org/wiki/Герри,_Элбридж_Томас Автор: Алексей Волчок ("Хронически разумные United") Вопрос 14: Известен курьезный случай, когда картина Анри Матисса "Лодка" провисела в нью-йоркском музее 47 дней вверх ногами, прежде чем ее догадались перевернуть. Назовите картину, которую в конце XX века предложил перевернуть Алексий II. Ответ: "Пловец". Комментарий: Картина Валерия Балабанова "Пловец" известна вам из четвертого вопроса. В 1994 году на месте бассейна "Москва" вновь началось возведение Храма Христа Спасителя. Произведение тоже на водную тематику, и отражение присутствует. Источник: 1. https://en.wikipedia.org/wiki/Le_Bateau 2. https://ru.wikipedia.org/wiki/Пловец_(картина) Автор: Герман Чепиков, Алексей Гончаров ("Хронически разумные United") Вопрос 15: "Морской бой" - это один из самых популярных советских игровых автоматов. Существовала даже его специальная версия для подводных лодок под названием "Электронный тренажер". Она отличалась отсутствием выдвигающейся подставки для детей и... Чего еще? Ответ: Прорези для монет. Зачет: Монетоприемника. Источник: 1. https://engineering-ru.livejournal.com/221481.html 2. https://relikva.com/@pgmuseum/r/igrovoy_avtomat_morskoy_boy Автор: Надежда Лейчинская ("Ять") Вопрос 16: В одном из положений итальянского избирательного закона 2015 года говорится о том, что молодые итальянцы, проживающие за пределами страны не менее трех месяцев с определенной целью, имеют право голосовать в так называемом "зарубежном округе". Это положение получило название "поправка ПРОПУСК". Заполните пропуск латинизированным именем. Ответ: Erasmus. Зачет: Эразмус. Комментарий: Одна из приведенных целей - учеба. "Erasmus" - это некоммерческая программа Европейского союза по обмену студентами и преподавателями между университетами стран членов Евросоюза, а также латинизированная форма имени Эразм. Источник: https://ru.wikipedia.org/wiki/Итальянский_избирательный_закон_(2015) Автор: Александр Морозов ("Хронически разумные United") Вопрос 17: В конце 1950-х годов в США активно искали способы использования ядерного оружия в мирных целях. Название одного из таких проектов можно перевести на русский язык устаревшим словом среднего рода. Назовите это слово. Ответ: Орало. Комментарий: Проект носил название "Plowshare", в переводе на русский - "лемех" или "орало" из библейской фразы "перековать мечи на орала". Источник: https://republic.ru/posts/71669 Автор: Надежда Лейчинская ("Ять") Вопрос 18: Дуплет. 1. На картинке в сообществе "Химик-психопат" ОН сообщает, что сделал тебя мастером высокоэффективной жидкостной хроматографии. Назовите ЕГО. 2. В ЕГО оригинальное название входит латинское наименование слона. Собственный корреспондент газеты "Аргументы и Факты" в Амстердаме утверждает, что ОН шершавый. Назовите ЕГО коротким словом. Ответ: 1. Кот [в колпаке и с волшебной палочкой, который делает "вжух"]. 2. Ждун. Комментарий: 1. ВЖХ - и ты мастер высокоэффективной жидкостной хроматографии. 2. Оригинал скульптуры, которая в оригинале называется "Гомункулос Лаксодонтус", находится в Амстердаме. В его имя входит название африканского слона, а внешний вид во многом позаимствован у морского. Источник: 1. https://vk.com/himik_psihopat?w=wall-29287308_291342 2. http://www.aif.ru/society/people/skulptura_stavshaya_memom_aif_navestil_zhduna Автор: Иван Топчий ("Хронически разумные United"), Валерий Семёнов ("Любовь Каксон") Вопрос 19: [Ведущему: обязательно прочитать комментарий!] Герой современного сериала, полицейский, наблюдает за напарником через камеры в здании и дает ему по рации указания, в том числе издевательские. Когда напарник начинает игнорировать эти приказы, герой упоминает ПАРК ЮРСКОГО ПЕРИОДА. Что мы заменили словами "ПАРК ЮРСКОГО ПЕРИОДА"? Ответ: "Мир Дикого Запада". Зачет: "Западный мир"; "Westworld". Комментарий: Герой шутит, что напарник - робот, вышедший из-под контроля, как в сериале "Мир Дикого Запада". Источник: Телесериал "Бруклин 9-9", s04e11. Автор: Иван Топчий ("Хронически разумные United") Вопрос 20: В аниме "Ergo Proxy" [Эрго прОкси] описывается эпидемия вируса "Кот", который дает роботам самосознание. Какие три буквы мы пропустили в предыдущем предложении? Ответ: г, и, о. Зачет: g, i, o. Комментарий: Вирус называется "КОгито" - одно из ключевых понятий в философии Декарта, обозначающее представление, мысль, желание. Знаменита фраза Декарта "Cogito ergo sum" [кОгито Эрго сум] - "мыслю, следовательно существую". Источник: 1. https://en.wikipedia.org/wiki/Ergo_Proxy 2. https://ru.wikipedia.org/wiki/Cogito Автор: Валентин Копочель ("Автостопом по Прибалтике") Вопрос 21: Цитата из книги Рона Гарана: "Майк, на счету которого было уже три ИКСА, решил, что должен дать мне какой-нибудь совет: "Ронни, не смотри вниз" - вот всё, что пришло ему в голову". Конец цитаты. 380-й по счету ИКС был совершен 21 декабря 2015 года. Какие несколько слов мы заменили ИКСОМ? Ответ: Выход в [открытый] космос. Комментарий: Попробуй еще разбери в космосе, где низ, где верх. Источник: 1. https://esquire.ru/ronald-garan 2. https://ru.wikipedia.org/wiki/Список_выходов_в_открытый_космос_с_351-го_(с_2012_года) Автор: Надежда Лейчинская ("Ять") Вопрос 22: Альбом "Limp Bizkit" [лимп бИскит] "Stampede of the Disco Elephants" [стэмпИд оф зэ дИско Элефантс] записывается с 2012 года. Англоязычная Википедия, говоря об альбоме, упоминает ЕЕ. Статья о возобновлении блокировки интернет-ресурсов после окончания Олимпиады называлась "ОНА закончилась". Назовите ЕЕ двумя словами. Ответ: Китайская демократия. Комментарий: Википедия проводит аналогию между альбомом "Limp Bizkit" и альбомом "Guns'n'Roses", которого фанатам пришлось ждать 12 лет. На время Олимпиады в Пекине по просьбе иностранных журналистов был открыт доступ к китайским версиям сайтов BBC и "ГОлоса Америки". Источник: 1. https://en.wikipedia.org/wiki/Stampede_of_the_Disco_Elephants 2. http://os.colta.ru/news/details/6559/ Автор: Павел Белькевич ("Автостопом по Прибалтике") Вопрос 23: (pic: 20161025.jpg) Напишите русское название книги, часть обложки которой перед вами. Ответ: "Игра в классики". Комментарий: (pic: 20161026.jpg) Источник: https://ru.wikipedia.org/wiki/Игра_в_классики_(роман) Автор: Герман Чепиков ("Хронически разумные United") Вопрос 24: Одна из современных теорий зарождения жизни предполагает появление первых самокопирующихся молекул РНК не в объеме водного раствора, как считали ранее, а в виде слоя на основании из глины. Такая модель носит название "ОНА". Назовите ЕЕ двумя словами, начинающимися на одну и ту же букву. Ответ: Первичная пицца. Комментарий: По аналогии с "первичным бульоном". Источник: https://theoryandpractice.ru/posts/14132-nikitin Автор: Иван Топчий ("Хронически разумные United") Вопрос 25: Филолог Павел БеркОв был арестован в 1938 году по ЛОЖНОМУ обвинению в шпионаже. В надежде на оправдание он сочинил ФАЛЬШИВЫЕ показания. На какой заграничной улице были расположены его ВЫДУМАННЫЕ явки? Ответ: Унтер ден Линден. Комментарий: На языке узников того времени обвинения Беркову "липили", вот и показания он дал липовые. Унтер ден Линден - улица "Под липами". Источник: http://scisne.net/a-1675 Автор: Елизавета Лысенко ("Не вопрос") Вопрос 26: <раздатка> It's art But Is It Art В названии статьи, посвященной критике современного романа, мы дважды пропустили одну букву. Назовите автора этого романа. Ответ: [Донна] Тартт. Комментарий: Не все критики считают нашумевший роман "Щегол", поднимающий тему значимости искусства, настоящим искусством. Слова в заголовках на английском языке всегда пишутся с большой буквы, это могло служить подсказкой. Источник: https://www.vanityfair.com/culture/2014/07/goldfinch-donna-tartt-literary-criticism Автор: Елизавета Лысенко ("Не вопрос") Вопрос 27: Индастриал-группа "Einstürzende Neubauten" [айнштЮрценде нойбАутен], следующая принципам дадаизма, в 1980-е годы не могла позволить себе профессиональные инструменты и сделала своим девизом фразу "Будь умнее, воруй ТАМ". Часть действия фильма 1965 года происходит ТАМ. Назовите этот фильм. Ответ: "Операция "Ы" и другие приключения Шурика". Комментарий: ТАМ - это на стройке. Один из принципов дадаизма - это использование готовых объектов в новом ключе (например, строительных инструментов как музыкальных). Источник: http://www.music-rock.ru/showthread.php?t=4311 Автор: Елизавета Лысенко ("Не вопрос") Вопрос 28: <раздатка> бестиарий Еженедельно слушатели Радио Минск получают возможность насладиться коллекцией известных песен и редких композиций в передаче "BESTиарий". В названии передачи на раздаточном материале мы заменили часть букв. Восстановите это название в исходном виде. Ответ: BESTиарий. Зачет: С любой капитализацией. Комментарий: Такая вот игра слов, оценить которую могут не все слушатели, а только те, кто посещал сайт радиостанции. Источник: http://www.radiominsk.by/Program/GetDetails?composerId=125 Автор: Александр Курзинер ("Не вопрос") Вопрос 29: В России XIX века Америка воспринималась как что-то очень далекое и практически несуществующее. Поэтому один из персонажей романа Достоевского сообщает, что собирается уехать в Америку, а затем ДЕЛАЕТ ЭТО. Какие слова мы заменили словами "ДЕЛАЕТ ЭТО"? Ответ: Совершает самоубийство. Зачет: Убивает себя, совершает суицид и т.п. по смыслу. Незачет: Умирает. Комментарий: Америка была настолько далеко, что "уехать в Америку" было равносильно выражению "отправиться на тот свет", что и делает Свидригайлов в конце "Преступления и наказания". Источник: https://magisteria.ru/silver-age/russkij-simvolizm/ Автор: Елизавета Лысенко ("Не вопрос") Вопрос 30: На одном форуме представительница определенной профессии рассказала, что не боится, а наоборот, очень любит ЕЕ, поскольку в это время ей ничего не нужно делать. Слово "ОНА" происходит от латинского прилагательного со значением "бурный". Мы не спрашиваем вас профессию женщины. Какое слово мы заменили словом "ОНА"? Ответ: Турбулентность. Комментарий: Профессия этой женщины - стюардесса. В периоды сильной турбулентности ей не нужно обслуживать пассажиров самолета. Источник: 1. https://fishki.net/2022873-chto-stjuardessy-dumajut-o-passazhirah-anonimnye-priznanija.html 2. https://ru.wikipedia.org/wiki/Турбулентность Автор: Павел Малецкий ("Хронически разумные United") Вопрос 31: В романе "Последний сейм Речи Посполитой" избранное общество наслаждается роскошным приемом, который заканчивается фейерверком. Увидев, что огни фейерверка начинают складываться в буквы, один из героев романа мрачно произносит несколько слов. Кому, как считается, эти слова были адресованы задолго до этого? Ответ: Валтасару. Комментарий: Аллюзия на библейский сюжет "Пир Валтасара", когда вавилонский царь беспечно пирует, не зная, что дни его царства сочтены. Слова, о которых идет речь в вопросе: "мене, текел, упарсин". Источник: В. Реймонт. Последний сейм Речи Посполитой. http://flibusta.is/b/472789/read Автор: Павел Митар ("Ять") Вопрос 32: В уже известном вам сообществе "Химик-психопат" была опубликована фотография монаршей семьи азиатской страны. Назовите эту страну. Ответ: Бутан. Комментарий: Вряд ли другая страна заинтересовала бы химиков. Источник: https://vk.com/himik_psihopat?w=wall-29287308_298505 Автор: Иван Топчий ("Хронически разумные United") Вопрос 33: В начале XIX века спички были не слишком удобны в использовании: они могли сильно вспыхивать, искрить и из-за своего химического состава неприятно пахли. Назовите имя с двумя латинскими корнями, от которого произошло сленговое название таких спичек. Ответ: Люцифер. Комментарий: Спички-люциферики несут свет и адски пахнут серой. Источник: 1. https://en.wikipedia.org/wiki/Match 2. https://ru.wikipedia.org/wiki/Люцифер Автор: Надежда Лейчинская ("Ять") Вопрос 34: (pic: 20161027.jpg) При написании картины "Третье мая 1808 года в Мадриде" Франсиско де Гойя решил ДАТЬ КАРТ-БЛАНШ одному из героев, пренебрегая правилами казней тех времен. Какие два слова, начинающиеся на одну и ту же букву, мы заменили словами "ДАТЬ КАРТ-БЛАНШ"? Ответ: Развязать руки. Источник: https://ru.wikipedia.org/wiki/Третье_мая_1808_года_в_Мадриде Автор: Герман Чепиков ("Хронически разумные United") Вопрос 35: В списках самых некрасивых зданий мира фигурирует и Национальная библиотека в Буэнос-Айресе. Назовите человека, ответственного за реализацию проекта ее постройки. Ответ: [Хорхе Луис] Борхес. Комментарий: На момент выбора проекта Борхес был уже почти слепым. Источник: https://realt.onliner.by/2016/04/16/brutal Автор: Николай Будник ("Zемляне") Вопрос 36: "Мир Дикого Запада" - фильм 1976 года по сценарию Майкла Крайтона, рассказывающий о том, как в высокотехнологичном развлекательном комплексе экспонаты-андроиды выходят из-под контроля и нападают на посетителей. Назовите фильм, снятый по сценарию Крайтона около двадцати лет спустя. Ответ: "Парк Юрского периода". Комментарий: Только вместо андроидов - динозавры. Замена в девятнадцатом вопросе была неслучайной. Источник: https://ru.wikipedia.org/wiki/Западный_мир_(фильм) Автор: Надежда Лейчинская ("Ять") Тур: 5 этап Дата: 18-Mar-2017 Редактор: Сергей Дубелевич (Минск) Инфо: Ведущие команды: "Middle", "Корпрусариум", "Легионеры Боливарии", "Фунтики", "Хунвейбины". Вопрос 1: [Ведущему: выделить слово "приветствует".] Необразованный герой повести Роберта Колотухина приветствует политику Гитлера. В конце своего обращения к жильцам дома он употребляет слово из трех букв. Какое? Ответ: "Хай". Комментарий: Вместо "Хайль Гитлер!" герой пишет: "Хай Гитлер!". "Хай" по-украински и по-белорусски - "пусть". Также по звучанию совпадает с английским "hi" - "привет". Источник: Р.В. Колотухин. Наш дом стоит у моря. http://flibusta.is/b/321080/read Автор: Сергей Дубелевич ("Middle") Вопрос 2: На сайте extreme-hunter.ru [экстрИм хАнтер точка ру] ОНА обойдется вам примерно в 200 долларов, однако 150 лет назад ОНА обошлась в 36 тысяч раз дороже. Назовите ЕЕ одним словом. Ответ: Аляска. Комментарий: В первом случае имелась в виду куртка. Источник: 1. http://www.extreme-hunter.ru/shop/odezhda/kurtki/alyaska/ 2. https://ru.wikipedia.org/wiki/Продажа_Аляски Автор: Иван Зайков ("Легионеры Боливарии") Вопрос 3: Житель индийского Телангана по имени Рамеш бросил колледж и устроился работать на фармацевтический завод. Через некоторое время его арестовали. На сайте Lenta.ru говорится, что Рамеш получил прозвище "теланганский Кларк Кент". Назовите имя и фамилию, которые мы заменили словами "Кларк Кент". Ответ: Уолтер Уайт. Комментарий: Рамеш выносил с фармацевтического завода необходимые препараты и варил дома амфетамин. За это он получил прозвище Теланганский Уолтер Уайт. Уолтер Уайт является главным героем сериала "Во все тяжкие". Имя одного героя мы заменили именем другого, которое также начинается на одинаковые буквы. Источник: https://lenta.ru/articles/2017/02/10/india_frugs/ Автор: Василий Бобков ("Корпрусариум") Вопрос 4: Героиня мультсериала "Барбоскины" вынуждена во всём угождать младшему брату. В одном из эпизодов она танцует. Ответьте тремя словами: что в это время делает малыш? Ответ: Играет на дудке. Зачет: Дудит в дудку; дует в дудку. Комментарий: Вероятно, обыгрывается выражение "плясать под чужую дудку". Источник: Мультсериал "Барбоскины", 31-я серия. Автор: Сергей Дубелевич ("Middle") Вопрос 5: Внимание, в вопросе есть замена. В первой половине XX века выпускалась ТАКАЯ косметика. Настоящую славу ей принес постер, на котором женщина буквально светится от средств марки. Что мы заменили словом "ТАКАЯ"? Ответ: Радиоактивная. Комментарий: Средства содержали одновременно бромид радия и хлорид тория - сразу два радиоактивных соединения. Кремы, пудры, помады и зубные пасты "Tho-Radia" продавались до начала 1960 годов, когда популярность косметики с радием наконец сошла на нет. Источник: https://daily.afisha.ru/beauty/3434-nogi-lotosy-kraska-ubiyca-i-drugie-uzhasy-mira-krasoty/ Автор: Елена Гордынец ("Легионеры Боливарии") Вопрос 6: Дуплет. 1. В одном из эпизодов сериала клиент Шерлока Холмса, прогуливаясь с ним по городу, замечает, что за ними наблюдают, и произносит фразу из произведения XX века. Назовите автора этого произведения. 2. "Аутсайдер супротив тоталитарщины" - так один из интернет-пользователей охарактеризовал основные события фильма "Альфавиль", события которого происходят в XX веке. А в каком году? Ответ: 1. [Джордж] Оруэлл. 2. 1984. Комментарий: 1. В сериале за Шерлоком Холмсом часто следит его брат Майкрофт. Клиент произносит фразу: "Большой брат следит за вами?". 2. "Альфавиль" - фильм-антиутопия, где город Альфавиль находится под контролем профессора фон Брауна. В городе объявлены вне закона все гуманные человеческие чувства. Источник: 1. https://www.afisha.ru/movie/166845/review/363000/ 2. https://kinanet.livejournal.com/1327442.html 3. Телесериал "Шерлок", s04e02, 20-я минута. Автор: Василий Бобков ("Корпрусариум") Вопрос 7: В вопросе есть замены. В фильме "Бегущий по лезвию" полицейский, описывая героиню-андроида, причастную к убийству людей, называет ее "Леди и бродягой". Какие слова мы заменили словами "леди" и "бродяга"? Ответ: Красавица и чудовище. Комментарий: Название одного голливудского мультфильма мы заменили названием другого. Источник: Х/ф "Бегущий по лезвию" (1981), реж. Ридли Скотт. Автор: Команда "Фунтики" Вопрос 8: <раздатка> - Так, значит, это и есть речка? - Не речка, а река, - поправил его дядюшка Рэт, - а точнее, Река с большой буквы, понимаешь? В оригинальной ответной реплике героя повести Кеннета Грэма "Ветер в ивах" было всего два слова. Назовите первое из них. Ответ: The. Комментарий: Таким образом переводчик передал смысл реплики с отсутствующим в русском языке артиклем the. В оригинале диалог звучал так: - So this is a river! - THE River. Источник: 1. К. Грэм. Ветер в ивах (перевод И.П. Токмаковой). http://flibusta.is/b/129573/read 2. https://books.google.ru/books?id=NsF6AgAAQBAJ&pg=PA9#v=onepage&q&f=false Автор: Сергей Дубелевич ("Middle") Вопрос 9: Норвежская тминная водка обрела популярность в середине XIX века, после того как оказалась никому не нужной в Австралии. В более известной аналогичной истории фигурирует остров. Какой? Ответ: Мадейра. Зачет: Мадера. Комментарий: Согласно легенде, мадера получила свой вкус благодаря тому, что партия вина пережила длительное путешествие из Португалии в Индию и обратно. На норвежской тминной водке благотворно сказалось длительное путешествие с двумя пересечениями экватора. Источник: Н.В. Будур. Эти странные норвежцы. http://flibusta.is/b/162378/read Автор: Сергей Дубелевич ("Middle") Вопрос 10: Персонажей сериала "Американская история ужасов" преследует множество призраков. Например, в доме то и дело появляются две медсестры, а в двери ломятся крестьяне, вооруженные сельскохозяйственным инструментом. Какой культовый фильм часто "цитируется" в сериале? Ответ: "Сияние". Комментарий: Две одинаково одетые медсестры и крестьянин, проламывающий двери топором, воспроизводят известные сцены из фильма Кубрика. Источник: Телесериал "Американская история ужасов: Роанок". Автор: Артем Пьянков ("Легионеры Боливарии") Вопрос 11: В вопросе есть замена. Калмыцкая загадка об ИКСЕ звучит так: "В чашке - пестрое мясо". Разновидности какого блюда дал название ИКС? Ответ: Яичница. Комментарий: Глазунья. Источник: 1. https://ru.wikiquote.org/wiki/Калмыцкие_загадки 2. https://ru.wikipedia.org/wiki/Яичница Автор: Виталий Захарик ("Middle") Вопрос 12: В фантастическом романе Артура Кларка строительство ЕГО потребовало от человечества основательной чистки накопившегося мусора. Примером ЕГО в мифологии можно считать Иггдрасиль, который прорастает через шесть миров. Назовите ЕГО двумя словами, которые начинаются на соседние буквы алфавита. Ответ: Космический лифт. Комментарий: Строительство космического лифта требовало очистки орбиты от космического мусора. Источник: 1. http://www.mirf.ru/science/kosmicheskiy-musor 2. https://ru.wikipedia.org/wiki/Космический_лифт Автор: Василий Бобков ("Корпрусариум") Вопрос 13: В вопросе есть замена. Гровер Кливленд в свою бытность шерифом однажды выполнял функции ИКСА. В чешском костеле Святой Варвары ИКСУ предназначалось отдельное и самое удаленное место от всех прихожан. Назовите ИКСА одним словом. Ответ: Палач. Комментарий: До того как стать президентом США, Гровер Кливленд работал шерифом города Эри, штат Нью-Йорк. Однажды он должен был либо сам исполнить смертный приговор, либо за 10 долларов нанять палача. Кливленд предпочел казнить заключенного сам. А в средние века в костелах палачу нельзя было молиться вместе с прихожанами. Источник: 1. http://usa-info.com.ua/interesnye-fakty-o-prezidentakh-ssha/presidents/facts-presidents 2. http://possward.blogspot.com.by/2014/12/what-where-when-2014-winter-game-4.html Автор: Команда "Хунвейбины" Вопрос 14: Герой одной из новелл "Декамерона", переодевшись священником, исповедует свою жену. Чтобы она не узнала своего мужа, тот, в числе прочего, предварительно СДЕЛАЛ ЭТО. Назовите того, кто ДЕЛАЛ ЭТО в IV веке до нашей эры. Ответ: Демосфен. Комментарий: Чтобы героиня не узнала героя по голосу, он набирает в рот камешков. То же самое делал и Демосфен, совершенствуя свои ораторские способности. Источник: 1. Дж. Боккаччо. Декамерон. http://flibusta.is/b/342759/read 2. https://ru.wikipedia.org/wiki/Демосфен Автор: Станислав Адаскевич (Вильнюс) Вопрос 15: Пришедший на съемку Владимир Познер был удивлен, когда ведущий не подошел к нему на рукопожатие, а публика не встретила аплодисментами. В этот момент на съемочной площадке можно было увидеть, например, медитирующего человека, а также ЕГО вместо оператора. Идея фильма под названием "ОН" возникла у режиссера после прогулки по Пятой авеню, известной своими бутиками. Назовите ЕГО одним словом. Ответ: Манекен. Комментарий: На Пятой авеню города Нью-Йорка, как известно, расположено множество разнообразных бутиков. Режиссеру показалось, что один из манекенов, стоящих на витрине, ожил - так и родилась идея фильма. В первом случае речь идет о вирусном флешмобе "Mannequin Challenge" [мАнекен чЕллендж], суть которого заключается в подражании манекенам некоторой группы людей, пока их снимают. В данном случае для придания комичности во время съемки "Челленджа" возле одной из камер стоял манекен. Источник: 1. https://www.youtube.com/watch?v=ElQ1SVKQiiE&t=9m05s 2. https://ru.wikipedia.org/wiki/Манекен_(фильм) Автор: Иван Сергиевич ("Корпрусариум") Вопрос 16: <раздатка> Suji wa dokushin ni kagiru Перед вам первоначальное название ЭТОГО. Прообраз современного ЭТОГО под названием "Числовая площадь" был впервые опубликован в "Dell Magazines" [делл мЭгэзинз]. Назовите ЭТО. Ответ: Судоку. Комментарий: Первоначально игра называлась так, как приведено на раздатке. Позже название сократили до "su doku". В журналах и газетах часто публикуют кроссворды и судоку. Источник: https://books.google.ru/books?id=bmiYDQAAQBAJ&pg=RA2-PT48#v=onepage&q&f=false Автор: Василий Бобков ("Корпрусариум") Вопрос 17: Макиавелли пишет, что юного государя важно обучить тому, что врага можно победить как при помощи умных законов, так и грубой силой, поэтому античные герои пользовались ЕГО услугами. Назовите ЕГО имя. Ответ: Хирон. Комментарий: В государе должно содержаться как человеческое, так и животное начало. Кентавр Хирон был учителем Ясона, Ахилла и многих других героев. Макиавелли также известен другой своей животной аналогией - о том, что государь должен содержать в себе черты льва и лисы. Источник: Н. Макиавелли. Государь. http://flibusta.is/b/506547/read Автор: Дмитрий Дорожко (Рига) Вопрос 18: Рассказывая о бедности кубинцев, Борислав Козловский отмечает, что местные уличные торговцы, известные как "фосфореро", не только продают спички, но и ДЕЛАЮТ ЭТО. Какие два слова, начинающиеся на одни и те же две буквы, мы заменили словами "ДЕЛАЮТ ЭТО"? Ответ: Заправляют зажигалки. Комментарий: А в остальном мире пластмассовые зажигалки считаются одноразовыми. Источник: Журнал "GEO", 2014, N 7. Автор: Сергей Дубелевич ("Middle") Вопрос 19: Дуплет. 1. Немецкий писатель Зигмунд Графф писал: "АЛЬФА - это политика, которую уже нельзя исправить". Назовите АЛЬФУ. 2. Немецкий писатель Зигмунд Графф писал: "БЕТА - это история, которую еще можно исправить". Назовите БЕТУ. Ответ: 1. История. 2. Политика. Источник: https://dic.academic.ru/dic.nsf/aphorism/1637/ Автор: Виталий Захарик ("Middle") Вопрос 20: У главного героя одного фантастического романа нет ничего лишнего: шесть рук, две ноги, три глаза, хвост, крылья. Он способен на большой скорости рассекать по воздуху и по суше. Неспроста настоящая фамилия этого героя - АЛЬФА. Назовите АЛЬФУ. Ответ: Бритва. Комментарий: Олег Бритва - главный герой фантастических произведений Александра Рудазова о существе Яцхен. Фраза "нет ничего лишнего" - небольшой намек на принцип "бритвы Оккама". Источник: 1. http://ru.anime-characters-fight.wikia.com/wiki/Яцхен 2. А.В. Рудазов. Три глаза и шесть рук. http://flibusta.is/b/47135/read 3. http://fb.ru/article/66328/britva-okkama-otsekaya-lishnee Автор: Команда "Хунвейбины" Вопрос 21: (pic: 20161028.jpg) Как называется заведение, где беседуют персонажи игры "Святые Картофелины, мы что, в космосе?!"? Ответ: Spacebar. Зачет: Пробел, space bar, space. Комментарий: Вывеска заведения стилизована под клавиши command и пробел. Источник: https://www.youtube.com/watch?v=spVnY3753Nw&t=1m26s Автор: Артем Пьянков ("Легионеры Боливарии") Вопрос 22: В одном из эпизодов сериала "Воздействие" герой по имени Чарльз проходит через череду необычных событий, считая, что это сон. Назовите двумя словами животное, которое фигурирует в названии этого эпизода. Ответ: Белый кролик. Комментарий: Героя эпизода "Дело о белом кролике" решили назвать так же, как звали Льюиса Кэрролла, - Чарльз Доджсон. Приключения Алисы в Стране чудес происходили в ее снах. Источник: 1. http://leverage.wikia.com/wiki/The_White_Rabbit_Job 2. http://ru.wikipedia.org/wiki/Алиса_в_Стране_чудес Автор: Василий Бобков ("Корпрусариум") Вопрос 23: В начале своего стихотворения Дмитрий Авалиани называет себя "ящеркой ютящейся эпохи", а по ходу произведения неоднократно употребляет церковную лексику. Какое слово, завершающее произведение, рифмуется со словом "горЯ"? Ответ: Алтаря. Комментарий: Все слова в этом стихотворении расположены в обратном алфавитном порядке. Концовка выглядит так: Земная жизнь еще дарит, горя, высокое блаженство алтаря. Источник: https://45parallel.net/dmitriy_avaliani/ya_yascherka.html Автор: Сергей Дубелевич ("Middle") Вопрос 24: (pic: 20161029.jpg) Прозвище тройки Тоффоли - Пирсон - Картер совпадает с названием фильма, в котором освещаются война во Вьетнаме, Уотергейтский скандал, сексуальная революция. Напишите это название из одного слова. Ответ: "Семидесятые". Комментарий: В 1970-е произошли все указанные события, а хоккеисты играют под номерами 70 - 73 - 77. Источник: 1. https://www.sports.ru/tribuna/blogs/hokkeinosti/1086710.html 2. https://www.kinopoisk.ru/film/94193/ Автор: Вадим Германенко (Рига) Вопрос 25: Бутан никогда официально не был ничьей колонией, поэтому в этом государстве нет ИКСА. Один из слоганов "ИКСА" - "Не стройте планы на август". Назовите ИКС двумя словами. Ответ: День независимости. Комментарий: Действие фильма "День независимости" происходит в канун дня независимости США, т.е. в начале июля. В Бутане, в отличие от многих государств, нет праздника "День независимости". Источник: 1. https://en.wikipedia.org/wiki/Independence_of_Bhutan 2. https://ru.wikipedia.org/wiki/День_независимости_(фильм,_1996) Автор: Сергей Дубелевич ("Middle") Вопрос 26: Автогонщик из повести Андрея Жвалевского и Евгении Пастернак объясняет спутнику-подростку схему автомобильного трюка, находясь за столиком в кафе. После объяснения он, усмехнувшись, вспоминает об ЭТОМ ЧЕЛОВЕКЕ, но тинейджер его не понимает. Назовите ЭТОГО ЧЕЛОВЕКА. Ответ: Чапаев. Комментарий: Для иллюстрации схемы гонщик расставляет предметы на столе, а потом говорит, что поступает, как Чапаев в фильме. Спутница автогонщика, не знакомая с советской культурой, не знает, кто это. Источник: А.В. Жвалевский, Е.Б. Пастернак. Пока я на краю. http://flibusta.is/b/469108/read Автор: Сергей Дубелевич ("Middle") Вопрос 27: В сериале "Конь БоДжек" антропоморфные животные живут и работают наравне с людьми. В одной из серий БоДжек Хорсмэн приходит на съемки телешоу, а камера, следуя за героем, постепенно знакомит зрителя со съемочной группой, плавно переходя от одного персонажа к другому. Ответьте словом с двумя английскими корнями: кем является невезучий помощник режиссера? Ответ: Бёрдмэн. Комментарий: Серия копирует манеру съемки "одним дублем", известную по фильмам "Бёрдмэн" и "Выживший". Источник: Мультсериал "Конь БоДжек", s02e08. Автор: Артем Пьянков ("Легионеры Боливарии") Вопрос 28: В конце XIX века дирижер Уолтер Дамрош сумел завести нового друга - крупного американского предпринимателя. Дамрош оказал на него такое сильное влияние, что бизнесмен стал жертвовать крупные суммы на развитие искусства. Напишите фамилию этого предпринимателя. Ответ: Карнеги. Комментарий: По инициативе Эндрю Карнеги в Нью-Йорке был построен крупный концертный зал, известный как Карнеги-холл. В отличие от другого Карнеги - Дейла, автора книги "Как завоевывать друзей и оказывать влияние на людей", Эндрю Карнеги сам подвергся чужому влиянию. Источник: 1. http://www.states-of-america.ru/new-york-city/dostoprimechatelnosti/karnegi-holl/ 2. https://ru.wikipedia.org/wiki/Карнеги,_Дейл Автор: Виталий Захарик ("Middle") Вопрос 29: Внимание, в вопросе есть замены. Комик Александр Бёресс в своих интервью говорил, что не любит свое довольно редкое имя еще со школы, потому что его ровесники ничего не слышали о полководце, зато знали о Доценте. Какие два имени собственных мы заменили в этом вопросе? Ответ: Ганнибал, Лектер. Комментарий: Комик Ганнибал Бёресс жалуется, что в школе сверстники не слышали о карфагенском Ганнибале, в честь которого он был назван, но знали Ганнибала Лектера, из-за чего и посмеивались над ним. Источник: https://www.youtube.com/watch?v=W_4dMWkg0rIe&t=4m22s Автор: Артем Пьянков ("Легионеры Боливарии") Вопрос 30: Персонаж современной повести Катажины Грохоли, увидев возобновление работ на замороженной стройке, удивляется: неужели снова ОНО? Назовите ЕГО несклоняемым словом. Ответ: Евро. Комментарий: Катажина Грохоля - польская писательница. Перед чемпионатом Европы по футболу 2012 года в Польше было построено много новых объектов. Источник: К. Грохоля. Хьюстон, у нас проблема. http://flibusta.is/b/412937/read Автор: Сергей Дубелевич ("Middle") Вопрос 31: В Вышнем Волочке есть каналы и дамбы. Неудивительно, что на привокзальной площади этого города находится ОН. Назовите ЕГО тремя словами, начинающимися на одну и ту же букву. Ответ: Памятник Петру Первому. Комментарий: После пребывания в Голландии Петр I решил построить каналы в России. На памятнике он одобряет схему проекта водной системы, подготовленную Михаилом Сердюковым. Источник: 1. https://ru.wikipedia.org/wiki/Вышний_Волочёк 2. https://www.votpusk.ru/country/dostoprim_info.asp?ID=15621 Автор: Иван Зайков ("Легионеры Боливарии") Вопрос 32: Говоря о засекреченности нахождения в СССР Бруно Понтекорво, сайт colta.ru [кОлта ру] пишет, что Понтекорво стал "ИКСОМ". Можно сказать, что один сирота стал "ИКСОМ" из-за страха окружающих. Назовите имя, данное этому сироте при рождении. Ответ: Том. Зачет: Томас Марволо Реддл. Комментарий: ИКС - "Тот-Кого-Нельзя-Называть". Выросшего в приюте Тома Реддла, известного также как Волан-де-Морт, в мире волшебников боялись до такой степени, что даже его имя, как правило, не произносили. Целых пять лет нахождение Понтекорво было под секретом. Источник: 1. http://www.colta.ru/articles/science/6410 2. https://ru.wikipedia.org/wiki/Волан-де-Морт Автор: Иван Сергиевич ("Корпрусариум") Вопрос 33: Согласно исследованиям астрофизиков, в верхних слоях атмосферы Сатурна разряды молнии периодически затрагивают молекулы метана, при этом высвобождаются атомы углерода. Эти атомы соединяются друг с другом и, продолжая сжиматься, начинают движение к каменному ядру планеты. Поэтому, по мнению журналиста Аси Гориной, на Сатурне можно увидеть ЕГО. Ивану обещали, что он тоже увидит ЕГО. Назовите ЕГО тремя словами. Ответ: Небо в алмазах. Комментарий: Углерод, продолжая сжиматься, превращается в алмазы. Героиня пьесы Чехова "Дядя Ваня" обещает заглавному персонажу, что они тоже увидят небо в алмазах. Источник: 1. http://www.vesti.ru/doc.html?id=1140230 2. http://www.bibliotekar.ru/encSlov/13/167.htm Автор: Виталий Захарик ("Middle") Вопрос 34: Внимание, в вопросе есть замены. Нарушая правила, нетрезвый герой современной песни утверждает, что правая НОГА интереснее левой. Какое слово мы заменили словом "НОГА"? Ответ: Педаль. Комментарий: Главный герой, злоупотребив алкоголем, садится за руль и, разумеется, предпочитает жать педаль газа, которая в автомобиле обычно расположена справа. Слово "педаль" происходит от латинского "pedis" [пЕдис] - "нога". Источник: 1. http://www.gl5.ru/noize_mc_jameson.html 2. https://ru.wiktionary.org/wiki/педаль Автор: Иван Сергиевич ("Корпрусариум") Вопрос 35: Персонаж повести Бориса Акунина, действие которой происходит в XIX веке, скептически отзывается о своих перспективах: "Так и пробегаешь всю жизнь, не заслужив хорошего чина". Какое прилагательное в его цитате мы заменили? Ответ: Классного. Комментарий: В данном случае это слово обозначает "относящегося к одному из классов табели о рангах". Источник: Б. Акунин. Особые поручения: Пиковый валет. http://flibusta.is/b/187523/read Автор: Сергей Дубелевич ("Middle") Вопрос 36: Внимание, в вопросе есть замена. Говоря о вероятном конфликте между Британской и Японской империями, Ниал Фергюсон сравнивает Британию с ТАКИМ ИМ. Какие два слова, начинающиеся на парные согласные, мы заменили словами "ТАКОЕ ОНО"? Ответ: Заходящее солнце. Комментарий: Фергюсон сравнивает возможный конфликт старой Британской и новой Японской империй со столкновением заходящего и восходящего солнца. Источник: Н. Фергюсон. Империя: чем современный мир обязан Британии. http://flibusta.is/b/378579/read Автор: Елена Гордынец ("Легионеры Боливарии") Тур: 6 этап Дата: 15-Apr-2017 Редактор: 1-18 - Вера Рабкина (Минск); 19-36 - Анастасия Балмакова (Минск) Инфо: Вера Рабкина выражает благодарность Михаилу Карпуку, Александру Марцинкевичу, Анастасии Балмаковой, Павлу Чернявскому, Виктории Чернявской, Павлу Полярушу, Евгению Миротину, а также командам "Енотики-7" и "Эталон этанола" (все - Минск) за ценные замечания и помощь в подготовке пакета. Анастасия Балмакова выражает благодарность Михаилу Карпуку, Дарье Соловей, Александру Марцинкевичу, Александре Ермалович, Елене Ваксман-Атроховой, а также командам "Енотики-7" и "Эталон этанола" (все - Минск) за ценные замечания и помощь в подготовке пакета. Вопрос 1: Особенность ИХ образа жизни связана со сложным способом пищеварения, отнимающим много энергии. Сотрудники заповедника в Коста-Рике рассказывают, как после каждого мытья вешают ИХ на просушку. Назовите ИХ. Ответ: Ленивцы. Комментарий: Питание ленивца и является основной причиной его "ленивости" - животное старается тратить поменьше энергии, так как почти вся она нужна ему для переваривания. При этом листья являются очень низкокалорийной пищей и дают крайне мало энергии. Источник: 1. https://www.factroom.ru/facts/12064 2. https://www.youtube.com/watch?v=q1mAGQAw3Oc Автор: Вера Рабкина ("Енотики-7") Вопрос 2: В романе Дэна Симмонса рассказывается о том, как Уилки Коллинз создавал образ одного из первых английских детективов - сержанта Каффа. Сначала Коллинз предположил, что хобби Каффа будет ОНО, но потом передумал в пользу выращивания роз. Назовите ЕГО одним словом. Ответ: Пчеловодство. Комментарий: Все английские детективы немного похожи. Источник: Д. Симмонс. Друд, или Человек в черном. http://flibusta.is/b/229505/read Автор: Никита Геер ("Енотики-7") Вопрос 3: Здание Центрального ЗАГСа города Киева за необычную форму получило прозвище ИКС. Местные так и говорят об этом ЗАГСе: "Как туда пошел - пропал!". Назовите ИКС двумя словами. Ответ: Бермудский треугольник. Комментарий: Здание имеет форму треугольника, а о многих молодых мужчинах так говорят: "Женился - пропал!". Источник: 1. http://wedding.ua/company/register/kiev/81/ 2. https://www.interesniy.kiev.ua/zags-bermudskiy-treugolnik/ Автор: Евгений Зайцев ("Умник") Вопрос 4: Один немецкий ресторан предлагает своим посетителям преодолеть особенно высокий бургер под названием АЛЬФА. На другой АЛЬФЕ нанесены примеры европейского стрит-арта. Назовите АЛЬФУ двумя словами. Ответ: Берлинская стена. Комментарий: Несмотря на то что бургер Берлинская Стена очень высокий, некоторым посетителям всё же удается его преодолеть, равно как и настоящую стену удалось преодолеть некоторым берлинцам. Источник: https://www.facebook.com/hamburgburger/photos/a.688653644482783.1073741828.688640321150782/1200039526677523/?type=3&theater Автор: Вера Рабкина ("Енотики-7") Вопрос 5: (pic: 20161030.jpg) Перед вами один из возможных вариантов ИКСА. Назовите ИКС двумя словами. Ответ: Флаг Марса. Комментарий: На розданном изображении мы видим, что Марс - четвертая планета от Солнца, которое изображено справа. Источник: https://hi-news.ru/space/kakim-by-mog-byt-flag-marsa.html Автор: Кирилл Севкович ("Эталон этанола") Вопрос 6: Недавно компания, выпускающая АЛЬФУ, решила обновить набор из восьми предметов. Как ни удивительно, она отказалась от исключительного права единолично принимать решение и провела голосование среди фанатов. В результате АЛЬФА пополнится резиновой уточкой, тираннозавром и пингвином. Что мы заменили АЛЬФОЙ? Ответ: Монополия. Комментарий: Компания "Hasbro" отказалась от монополии на выбор новых фигурок и провела голосование. Источник: https://joinfo.ua/sociaty/1200833_Tirannozavr-rezinovaya-utochka-pingvin-Monopoliya.html Автор: Надежда Потрихалина ("Енотики-7") Вопрос 7: В книге Джареда Даймонда описывается сцена пленения Атауальпы отрядом Франсиско Писарро. В ней упоминается имя Сантьяго. Автор вопроса предположил, что в версии, адаптированной для современного американского читателя, могло бы присутствовать другое имя. Какое? Ответ: Джеронимо. Комментарий: Отряд Писарро использовал испанский боевой клич "Сантьяго!". Интересно, что клич "Джеронимо!", который также является именем, используется в США десантниками-парашютистами во время прыжка из самолета. Источник: Дж. Даймонд. Ружья, микробы и сталь. Судьбы человеческих обществ. http://flibusta.is/b/237834/read Автор: Никита Геер ("Енотики-7") Вопрос 8: Согласно одной из версий, современный вариант ЕЕ появился в 1904 году под названием "ОНА для холостяков" и предназначался для мужчин, которые не хотели пришивать пуговицы. В английском языке она получила название от АЛЬФЫ. Что мы заменили АЛЬФОЙ? Ответ: [Буква] T. Комментарий: Речь идет о майке с короткими рукавами, также известной как T-shirt [ти-шот]. Удивительно, но ее современный вариант появился только в 1904 году под названием "майка для холостяков". Рекламный слоган гласил: "Никаких булавок, пуговиц, ниток и иголок", а главной целевой аудиторией были холостые мужчины, которым некому было пришить оторвавшиеся пуговицы. Источник: https://www.nytimes.com/2013/09/22/magazine/who-made-that-t-shirt.html Автор: Вера Рабкина ("Енотики-7") Вопрос 9: В 1950-е годы в СССР остро стояла проблема давки на стадионах после окончания матча. Тогда организаторы пошли на хитрость, сделав ЕГО еще и ТАКИМ. Это в большой степени решило проблему, а также мотивировало зрителей оставаться до конца матча. Что мы заменили словом "ТАКИМ"? Ответ: Лотерейным. Комментарий: После матча по очереди для каждой трибуны разыгрывались призы. В качестве лотерейного выступал входной билет на матч. Источник: Журнал "Большой", октябрь 2016 г. Автор: Вера Рабкина ("Енотики-7") Вопрос 10: В тексте сценария ЭТО было помечено так: "Крылова приветствует гостей". В окончательном варианте ЭТО длится 222 секунды. Сколько ЭТО длилось, по словам самой Крыловой? Ответ: Пять минут. Комментарий: Имеется в виду сценарий фильма Рязанова "Карнавальная ночь", согласно которому героиня Гурченко Леночка Крылова пела приветственную песню. Длительность оригинального трека этой песни составляет 3 минуты и 42 секунды (или 222 секунды). Песня называется "Пять минут". По тексту песни "... но, пока я песню пела, пять минут уж пролетело...". Источник: 1. http://www.radioshanson.fm/news/istoriya_pesni__pyat__minut 2. https://zf.fm/song/2243 Автор: Евгений Зайцев ("Умник") Вопрос 11: ТАКОЙ ОН в произведении 1925 года является символом надежды, недосягаемым ориентиром для главного героя. Клятва ТАКОГО ЕГО содержала следующие слова: "И должен я пролить мой свет на темное зло, ибо тьма не устоит перед светом...". Что мы заменили словами "ТАКОЙ ОН"? Ответ: Зеленый фонарь. Комментарий: Зеленый цвет в литературе традиционно считается символом надежды. В романе "Великий Гэтсби" зеленый фонарь маяка имеет символический подтекст. Зеленые Фонари из одноименного комикса приносили клятву бороться со злом с помощью Зеленого фонаря. Источник: 1. http://fitzgerald.narod.ru/critics-rus/itkina-gatsby.html 2. https://ru.wikipedia.org/wiki/Зелёный_Фонарь Автор: Надежда Потрихалина ("Енотики-7") Вопрос 12: Английская инструкция 1672 года упоминает кварту воды, а также рекомендует подержать АЛЬФУ над паром. АЛЬФУ в классическом мультфильме озвучивал девятилетний Брэдли Пирс. Назовите АЛЬФУ двумя словами, начинающимися на одну и ту же букву. Ответ: Чайная чашка. Зачет: Чашка чая. Комментарий: Это инструкция по завариванию чая. Британцы еще в XVII веке нашли способ сберечь особенно ценный в ту эпоху фарфор. Брэдли Пирс озвучил Чипа - превращенного в чайную чашку мальчика из мультфильма "Красавица и Чудовище". Источник: 1. https://www.thevintagenews.com/2016/09/01/priority-act-drinking-tea-uk-introduced-1662-portuguese-queen/ 2. https://www.imdb.com/name/nm0682300/ Автор: Надежда Потрихалина ("Енотики-7") Вопрос 13: Прослушайте цитату: "Недавно читал книгу про ПРОПУСК. Жуткое, отвратительное чтиво. Но под конец мне даже начало нравиться". Заполните пропуск двумя словами. Ответ: Стокгольмский синдром. Источник: https://www.reddit.com/r/Jokes/comments/2d4v7o/i_just_read_a_book_about_stockholm_syndrome/ Автор: Вера Забавская Вопрос 14: <раздатка> Авиакомпания "Etihad Airways" запустила новый сервис "Etihad Wi-Fi" - широкополосный доступ в Интернет и мобильную связь на борту самолетов. Система разработана компанией "Panasonic" - одним из мировых лидеров среди поставщиков систем развлечения и связи в полете. В розданном отрывке мы слегка видоизменили одно слово. Напишите его в исходном варианте. Ответ: Wi-Fly. Зачет: Fly. Комментарий: Компания "Etihad" [этихАд] гордится тем, что предоставляемый вайфай доступен во время полета, что и подчеркнула в названии. Источник: https://www.airlines-inform.ru/news/etihad-wi-fly.html Автор: Вера Рабкина ("Енотики-7") Вопрос 15: [Ведущему: отточия в вопросе голосом никак не выделять.] Прослушайте отрывок из книги Наринэ Абгарян: "Она спрятала портрет за большим деревянным ларем <...>, где он с тех пор лежал <...>, безнадежно отсыревший и выцветший за ПРОПУСК, на которые его обрекли равнодушные дальние потомки". Заполните пропуск названием произведения 1967 года. Ответ: Сто лет одиночества. Источник: 1. Н.Ю. Абгарян. С неба упали три яблока. http://flibusta.is/b/498016/read 2. https://ru.wikipedia.org/wiki/Сто_лет_одиночества Автор: Вера Рабкина ("Енотики-7") Вопрос 16: Современная художница Мун Рибас в качестве арт-проекта вживила в свои ступни специальные микрочипы. Она отмечает, что некоторые ее знакомые были буквально потрясены ее решением. Когда эти чипы вибрируют? Ответ: Когда где-то в мире происходит землетрясение. Зачет: Когда приближается землетрясение и другие близкие по смыслу ответы, содержащие слово "землетрясение" или "сейсмическая активность". Комментарий: С помощью этих микрочипов Мун Рибас ощущает все землетрясения мира, и ее тело трясется вместе с другими жителями Земли, находящимися в эпицентре катастрофы. Источник: https://realnoevremya.ru/articles/43899 Автор: Вера Рабкина ("Енотики-7") Вопрос 17: Александр Мещеряков отмечает, что ОНО не устоялось в японской культуре, так как с его помощью нельзя выразить разную степень уважения. Назовите ЕГО одним словом. Ответ: Рукопожатие. Комментарий: Рукопожатие подразумевает, что все равны, в то время как поклон, принятый в Японии, может быть разной степени глубины. Источник: https://arzamas.academy/courses/21/4 Автор: Вера Рабкина ("Енотики-7") Вопрос 18: <раздатка> "История Древнего Рима за 20 минут" Перед вами название видеоролика, посвященного истории Древнего Рима. Что мы заменили в этом названии? Ответ: XX. Комментарий: Чтобы подчеркнуть тематику, авторы ролика заменили арабские цифры римскими. Источник: http://arzamas.academy/likbez/antiquity Автор: Дмитрий Медведев ("Енотики-7") Вопрос 19: БАджо - это народ в Юго-Восточной Азии. Многим его представителям еще в детстве прокалывают барабанные перепонки. Те, кто подвергся этой процедуре, впоследствии зарабатывают больше, чем их соплеменники. Назовите основной промысел бАджо. Ответ: Ловля жемчуга. Зачет: Добыча жемчуга; ловля/добыча морских огурцов; по смыслу. Комментарий: Баджо живут в домах на сваях и зарабатывают добычей жемчуга. Человек, которому с детства повредили барабанные перепонки, может без боли нырять на глубину до тридцати метров и добывать гораздо больше жемчуга. Источник: http://www.factroom.ru/facts/9890 Автор: Дарья Соловей ("Одушевленные аэросани") Вопрос 20: В романе, действие которого происходит в начале XX века, надзиратели разработали новый вид кандалов. Кого они пригласили для испытания своего изобретения? Ответ: [Гарри] Гудини. Комментарий: Гудини прославился сложными трюками с побегами и освобождениями. Источник: 1. Э. Доктороу. Рэгтайм. http://flibusta.is/b/491173/read 2. https://ru.wikipedia.org/wiki/Гарри_Гудини Автор: Александр Марцинкевич ("Одушевленные аэросани") Вопрос 21: В 1994 году в одной стране проводился конкурс семейных автомобилей. По условиям конкурса каждый автомобиль должен был комплектоваться неким устройством, причем только одним. Мы не просим вас назвать это устройство. Ответьте, в какой стране проводился этот конкурс. Ответ: Китай. Зачет: КНР; Китайская Народная Республика. Комментарий: Конкурс семейных автомобилей объявило китайское правительство в соответствии с китайской государственной программой "Одна семья - один ребенок", поэтому и детское автокресло должно было быть в автомобиле только одно. Источник: https://www.drive2.ru/b/1911467/ Автор: Евгений Зайцев ("Умник") Вопрос 22: С тех пор как в Китае стало модным пить дорогое вино, там можно купить бутылку настоящего "ЛафИт-РОтшильд" за 200-300 евро. Какое слово мы пропустили в предыдущем предложении? Ответ: Из-под. Зачет: От; использованную; выпитую; пустую; по смыслу. Комментарий: 200-300 евро за бутылку такого вина - это очень дешево. Купив же пустую бутылку из-под элитного вина и наполнив ее любым другим, китаец может пустить пыль в глаза другим китайцам. Источник: Журнал "GEO", 2016, N 1. Автор: Анастасия Балмакова ("Одушевленные аэросани") Вопрос 23: В фильме Нила Бломкампа в неком здании герой видит сувениры в виде одного из представителей отряда приматов. Назовите это здание. Ответ: Эмпайр-стейт-билдинг. Комментарий: Сражение с гигантской гориллой Кинг-Конгом в одноименном фильме происходило на крыше Эмпайр-стейт-билдинг. Источник: 1. Х/ф "Элизиум: Рай не на Земле" (2013), реж. Нил Бломпкамп. 2. https://ru.wikipedia.org/wiki/Кинг-Конг_(фильм,_1933) Автор: Александр Марцинкевич ("Одушевленные аэросани") Вопрос 24: В вопросе есть замена. По форме расторОпша похожа на зерна, но растет быстрее. Отсюда, по одной из версий, и происходит ее русское название. Какое слово мы заменили словом "расторОпша"? Ответ: СпорыньЯ. Комментарий: СпорыньЯ - род грибов, паразитирующий на злаках. Когда спорынья заражает злаки, в колосьях пораженных растений вместо зерен образуются твердые рОжки, по форме похожие на зерна и растущие быстрее них - "споро", отсюда и название. Расторопша - это тоже растение. Источник: https://drugoj-m.livejournal.com/27362.html Автор: Анастасия Балмакова ("Одушевленные аэросани") Вопрос 25: Для того чтобы происходил фотоэффект, фотоны должны иметь достаточно высокую энергию. Если фотоны имеют низкую энергию, электроны перестают вылетать из металла, и фотоэффект прекращается. Рассказывая об этом, Майкл Файер прибегает к аналогии с детской игрой. Одно из названий этой игры совпадает с именем персонажа. Какого? Ответ: Али-Баба. Комментарий: В этой игре группа детей из одной команды растягивается в шеренгу, держась за руки. Игрок из другой команды с разбегу бросается на эту шеренгу и, если бежит достаточно быстро (имеет высокую энергию), разрывает ее и продолжает двигаться, хотя и медленнее. При несколько меньшей скорости он всё еще может прорвать шеренгу. Однако если он будет бежать достаточно медленно, то не сможет пробиться сквозь нее, поскольку энергии не хватит, чтобы преодолеть энергию связи в шеренге. Али-Баба - герой арабской сказки "Али-Баба и сорок разбойников". Источник: 1. М. Файер. Абсолютный минимум. Как квантовая теория объясняет наш мир. http://flibusta.is/b/430636/read 2. https://ru.wikipedia.org/wiki/Али-Баба Автор: Анастасия Балмакова ("Одушевленные аэросани") Вопрос 26: Подвид одного соревнования включает выполнение таких фигур, как бабочка, океанская волна, карусель и обручальное кольцо. Ответьте несклоняемыми словами: что это за соревнование и с помощью чего выполняются эти фигуры? Ответ: Родео, лассо. Комментарий: Сегодня родео - это серьезный вид спорта, который включает целый ряд дисциплин, в том числе различные трюки с лассо. Источник: https://www.popmech.ru/adrenalin/10393-bremya-nastoyashchikh-kovboev-russkoe-rodeo/ Автор: Команда "Одушевленные аэросани" Вопрос 27: [Ведущему: в последнем предложении четко произнести слово "фамилии" во множественном числе.] Рассуждая о непринятии обществом генетически модифицированных организмов, Александр ПАнчин пишет о необходимости появления двух тезок от мира фермерства и генной инженерии, которые подарили бы миру такой трансгенный продукт, что его захотел бы надкусить каждый. Назовите фамилии этих тезок. Ответ: Джобс, ВОзняк. Комментарий: Надкушенное яблоко - логотип компании "Apple" [эпл], основанной двумя Стивами - Джобсом и ВОзняком. Известна фраза Джобса о том, что "люди не понимают, что им на самом деле нужно, пока сам им этого не покажешь". Воображаемые фермер вроде Стива Джобса и генный инженер вроде Стива Возняка могли бы подарить миру такое трансгенное яблоко, которое захотел бы надкусить каждый. Источник: 1. А.Ю. Панчин. Сумма биотехнологии. Руководство по борьбе с мифами о генетической модификации растений, животных и людей. http://flibusta.is/b/500145/read 2. https://ru.wikiquote.org/wiki/Стив_Джобс Автор: Александр Круглик ("Эталон этанола") Вопрос 28: Кубинская культура представляет собой смешение индейских, испанских, африканских и азиатских элементов. Антрополог Фернандо ОртИс сравнил кубинскую культуру с ахиАко. Ответьте односложным словом: что такое ахиАко? Ответ: Суп. Комментарий: ОртИс говорит, что кубинская культура подобна хорошему кубинскому супу ахиАко, где смешаны разнообразные традиции, обычаи и вкусы. Источник: http://www.mid.ru/ru/maps/cu/-/asset_publisher/ZCoR8WfDPJng/content/id/331612 Автор: Команда "Одушевленные аэросани" Вопрос 29: Виктору МартинОвичу движение ИХ напомнило чаинки в чашке, в которой только что размешали сахар. ИХ можно увидеть на картине 1890 года. Назовите ИХ точно. Ответ: ВорОны. Комментарий: Движение кружащихся в небе ворон Мартинович сравнил с движением чаинок в чашке. В 1890 году Винсент ван Гог написал "Пшеничное поле с ворОнами". Источник: 1. В.В. Мартинович. Сфагнум. http://flibusta.is/b/328933/read 2. http://ru.wikipedia.org/wiki/Пшеничное_поле_с_воронами Автор: Александр Марцинкевич ("Одушевленные аэросани") Вопрос 30: На советской карикатуре, высмеивающей хиппи, парень и девушка смотрят на картину с изображением ИХ. Какой художник однажды заплатил крестьянину, чтобы тот не трогал ИХ? Ответ: [Клод] Моне. Комментарий: (pic: 20161031.jpg) Карикатура высмеивает лохматые прически хиппи, которые на вид не слишком отличаются от изображенных на картине стогов сена. Кисти Клода Моне принадлежит серия из нескольких десятков картин, на которых стога сена изображены в разное время суток и при разной погоде. Однажды Моне даже заплатил крестьянину, чтобы тот не убирал сено в положенное время. Источник: 1. http://modagid.ru/articles/4625 2. М. де Декер. Клод Моне. http://flibusta.is/b/326460/read 3. http://www.liveinternet.ru/users/2010239/post106324013 Автор: Команда "Одушевленные аэросани" Вопрос 31: Сюрстрёмминг - шведский деликатес из консервированной квашеной сельди. Автор одной поваренной книги насмешливо писал, что на банкете сюрстрёмминг подадут лишь в двух случаях: если хозяин пожелает есть в одиночестве или если позовет гостей ПРОПУСК. А кто оказался ПРОПУСК в произведении XIX века? Ответ: [Коллежский асессор / майор] [Платон Кузьмич] Ковалёв. Комментарий: Пропущенные слова - "без носа". В процессе приготовления сюрстрёмминга при брожении образуется множество сильнопахнущих веществ, в том числе сероводород. Иногда даже в шутку советуют пробовать это блюдо, надев противогаз. В повести Николая Васильевича Гоголя "Нос" майор Ковалёв оказался на некоторое время без носа. Источник: 1. https://ru.wikipedia.org/wiki/Сюрстрёмминг 2. https://ru.wikipedia.org/wiki/Нос_(повесть) Автор: Анастасия Балмакова ("Одушевленные аэросани") Вопрос 32: На памятнике в Нью-Йорке, установленном 125 лет назад, можно увидеть три однотипных объекта. Напишите название любого из этих объектов. Ответ: "Санта-Мария". Зачет: "Пинта"; "Нинья". Комментарий: В год 400-летия открытия Америки (в 1892 году) был установлен памятник Колумбу на площади Коламбус-Сёркл. Причем это не просто памятник, а своего рода ростральная колонна - на ней в качестве элементов присутствуют носы трех кораблей Колумба. В предыдущем вопросе носа не было, а здесь целых три! Источник: http://www.vanderkrogt.net/statues/object.php?webpage=CO&record=usny28 Автор: Анастасия Балмакова ("Одушевленные аэросани") Вопрос 33: [Ведущему: объявить, что ответы сдаются на раздаточном материале, и попросить команды написать на нем игровые номера.] (pic: 20161032.jpg) Слева - альтернативное написание гласных искусственного языка волапЮк, предложенное его создателем ЙОханном ШлЕйером. Дорисуйте то, что мы заретушировали на раздатке. Ответ: (pic: 20161033.jpg) Комментарий: Язык волапЮк [Volapük] в своем написании на волапЮке также содержит ü [у-умлЯут]. Разработавший его немец ЙОханн ШлЕйер позаимствовал для нового языка умляутированные гласные из родного немецкого языка. Правда, альтернативное написание не прижилось и используется достаточно редко. Источник: https://ru.wikipedia.org/wiki/Волапюк Автор: Анастасия Балмакова ("Одушевленные аэросани") Вопрос 34: В книге Евгения Клюева можно встретить подразделы "выступление", "наступление", "преступление", "исступление". Какое слово мы несколько раз пропустили в предыдущем предложении? Ответ: Лирическое. Комментарий: Писатель Евгений Клюев в своей книге "Между двух стульев" очень любит играть словами. По аналогии с лирическим отступлением автор делает "лирическими" и слова с другими приставками. Источник: Е.В. Клюев. Между двух стульев. http://flibusta.is/b/67087/read Автор: Анастасия Балмакова ("Одушевленные аэросани") Вопрос 35: В эпоху плейстоцена остров был покрыт льдом, а когда ледник растаял, остров уже был отделен от других земель проливом. Так ученые объясняют то, что легенда приписывает ЕМУ. Назовите ЕГО. Ответ: [Святой] Патрик. Комментарий: Таково научное объяснение отсутствия змей в Ирландии. На льду они жить не могли, а потом не смогли переплыть пролив. Некоторые люди считают, что змей из Ирландии изгнал лично святой Патрик. Источник: http://www.vokrugsveta.ru/quiz/263387/ Автор: Команда "Одушевленные аэросани" Вопрос 36: Рассказывая о том, что трилобиты вымерли не все сразу, а, так сказать, "ушли со сцены" эволюции постепенно, палеонтолог Ричард ФОрти упоминает австрийца. Назовите этого австрийца. Ответ: [Йозеф] Гайдн. Комментарий: Игравшие важную роль в палеозое членистононие организмы трилобиты вымирали постепенно. Это был длительный процесс, а не результат некоего природного катаклизма или массового вымирания видов. Автор книги "Трилобиты: свидетели эволюции" Ричард Форти упоминает в связи с этим "Прощальную симфонию" Йозефа Гайдна, в финальной части которой музыканты постепенно, один за другим, прекращают играть, гасят свечи и покидают сцену. Это был последний вопрос, и мы тоже прощаемся с вами. Источник: 1. Р. Форти. Трилобиты. Свидетели эволюции. http://flibusta.is/b/363864/read 2. https://ru.wikipedia.org/wiki/Симфония_%E2%84%96_45_(Гайдн) Автор: Анастасия Балмакова ("Одушевленные аэросани")